2003-2010年全国大学生英语竞赛C类初赛、决赛真题及答案汇总集(原创)(1)

发布时间:2019-10-21 20:13:48

2003年全国大学生英语竞赛初赛试题

2003 National English Contest for College Students

Preliminary

Part I Listening Comprehension30 minutes30 points

Section A Dialogues10 points

DirectionsIn this section you will hear 10 short dialoguesAt the end of each dialoguea question will be asked about what was saidBoth the dialogue and the question will be read only onceAfter each question there will be a pauseduring the pauseyou must read the four choices marked ABC and Dand decide which is the best answerThen m ark the corresponding letter on the Answer Sheet with a single line through the center

1AA sales clerk BA police officer

CA tailor DA nurse

2ABy train BShe walks

CBy car DBy bus

3AFish is the only dish left BChicken is the only dish left

CVegetarian meals are not offered DThere aren't any vegetarian meals left

4AHe starts work next weekend BHe'll be away

CHe'll be in the mountains DHe's moving to Florida

5AIn an elevator BAt a dress store

COn the seventh floor DAt a department store

6AThey felt it was disorganized

BThey were pleased with its Asian content

CThey felt it lacked Asian content

DThey felt it ignored recent events

7AHe doesn't have enough time

BHe doesn't have a watch

CThe library doesn't have the articles he wants

DHe can't find the library

8AHe wants the woman to dine out with them

BHe wants to work tomorrow

CHe wants the woman to finish dinner first

DHe wants to pay for the dinner

9ATwice a day BTwice a week

COnce a week DDaily

10AAt two o'clock BAt four o'clock

CAt three thirty DAt eight o'clock

Section B News Items10 points

DirectionsIn this sectionyou will hear 10 pieces of short news from BBC or VOAThere will be a question following each piece of newsWrite down the answer to each question in no more than 15 words

11_______________________________________

12_______________________________________

13_______________________________________

14_______________________________________

15_______________________________________

16_______________________________________

17_______________________________________

18_______________________________________

19_______________________________________

20_______________________________________

Section C Compound Dictation10 points

DirectionsIn this sectionyou will hear a passage three timesWhen the passage is read for the first timeyou should listen carefully for its general ideaThen listen to the passage againWhen the passage is read for the second timeyou are required to fill in the blanks numbered from 21 to 28 with the exact words you have just heardFor blanks numbered from 29 to 30you are required to fill in the missing informationYou can either use the exact words you have just heard or write down the main points in your own wordsFinallywhen the passage is read for the third timeyou should check what you have written and rewrite the correct answers on the Answer Sheet

Although general Motors and General Electric are large multinational companies with operations around the globethere are numerous smaller companies that engage in international tradeBecause 95percent of the world's population and two-thirds of its (21)_____ power are located outside the United Statesit is important for American (22)_____to be present in foreign marketsHoweverbefore we explain the different methods by which a company may (23)_____in international tradewe might first consider some important (24)_____that U.Scompanies often fail to study before they sell products in a foreign countryThese factors are (25)_____with differences in languagein values and attitudesand in political (26)_____

When (27)_____Coca-Cola into the Chinese market in 1920the company used a group of Chinese symbols thatwhen spokensounded like Coca-ColaHoweverwhen read these symbols meant,“a female horse fattened with wax”.Upon reentering the Chinese market in the 1970sCoca-Cola used a series of Chinese (28)_____that translates intohappiness in the mouth”.(29)_________________________

Culture is the total pattern of human behavior that is practiced by a particular group of people(30)_________________________

Part II Vocabulary and Structure15 minutes30 points

Section A Multiple Choice20 points

DirectionsQuestions 31-50 constitute a complete passageThere are 20blanks in the passageFor each blank there are four choices marked ABC and DChoose the one that best completes the sentenceThen mark the corresponding letter on the Answer Sheet with a single line through the centre

31Senior Metropolitan police officers tried to dismiss the Noting Hill race riots which raged for five nights over the August bank holiday in 1958 as the work ofruffians both colored and whitehell-bent on hooliganism according to _____ official files

Arecent revealed Bnewly released

Cprevious disclosing Dearlier exposing

32But police eyewitness reports in the secret papers_____ that they were overwhelmingly the work of a white working class mob out to get theniggers”.

Acontain Bconvince

Cconsist Dconfirm

33The ferocity of the Noting Hillracial riots”,as the press called them at the timeshocked Britain into_____ for the first time that it was not above the kind of racial conflict then being played out in the American deep south

Arealizing Bwitnessing

Cwatching Didentifying

34The carnivalwhich will_____ the streets of west London _____more than 1.5 million people this weekendwas started in 1959 as a direct response to the riots

Acrowdof Bpourfor

Cfillwith Demergein

35While senior officers tried to play down the racial aspects of the riotsthe internal Metropolitan police files released this month at the public record office confirm that the disturbances were overwhelmingly _____ by 300 to 400 strongKeep Britain White mobs many of them Teddy boys armed with iron bars butcher's knives and weighted leather beltswho wentnigger-huntingamong the West Indian residents of Noting Hill and Noting Dale

Aerupted Bcommenced

Ctriggered Dinaugurated

36The first night left five black men _____ on the pavements of Noting Hill

Alying unconscious Bthere died

Cfeel faint Dserious hurt

37The battles raged over the bank holiday weekend as the black _____responded in kind with counterattacks by large groups ofmen of colorsimilarly armed

Acolumn Barmy

Cbrigade Dcommunity

38Thomas Williams was stopped by the police as he came out of Bluey's Club on Talbot RoadNoting HillHe _____a piece of iron down his left trouser lega petrol bomb in his right pocket and a razor blade in his inside breast pocket:“I have to protect myself,”he told the arresting officer

Afound to have Bwas found to have

Cfound having Dwas found having

39The _____ fileswhich were sealed under the 75-year rule but have been released earlyshow that senior officers tried to convince the then home secretary,“RabButlerthat there was not a racial element to the rioting

Aforbidden Bconfidential

Cincredible Dstrict

40In his official reportDetective Sergeant M.Walters of the Notting Hill police said the national press had been wrong to portray thewidespread series of street disturbancesasracialriots:“Whereas there certainly was some _____ feeling between white and colored residents in this areait is abundantly clear much of the trouble was caused by ruffiansboth colored and whitewho seized on this opportunity to indulge in hooliganism .”

Aill Bsick

Cpainful Dhurt

41But the police witness statements and private statistics _____

Atold differently Binterpreted in a different way

Cexisted m any differences Dtold a different story

42The Met com missioner was told that _____ the 108people who were charged with offences ranging from grievous bodily harm to affray and riot and possessing offensive weapons72 were white and 36 were colored”.

Afor Bfrom

Cof Din

43It is popularly believed that the riot began on the night of SaturdayAugust 20when a 400-strong crowd of white men_____Teds”,attacked houses occupied by West Indians

Athey are all Bmany of them

Csome were Dmost of them belong to

44Among the _____ was Majbritt Morrison a young white Swedish bride of a Jamaican

Aoffenders Brioters

Cvictims Dresidents

45She was pelted with stonesglass and woodand _____ in the back with an iron bar as she tried to get home

Abruised Bstruck

Cpatted Dscratched

46The internal police witness statements provide graphic evidence of the motives of the mobs—at one point crowds several thousand strong roamed the streets of Notting Hill_____ homes and attacking any West Indian they could find

Aplunging into Bbreaking into

Cseeking for Dsearching for

47PC Richard Bedford said he had seen a mob of 300 to 400 white people in Bramley Road _____:“We will kill all black bastardsWhy don't you send them home?”

Ashouting Bto cry

Cutter Dannounced

48PC Ian McQueen on the same night said he was told:“Mind your own _____copsKeep out of itWe will settle these niggers our wayWe'll murder the bastards.”

Amatters Baffair

Cthings Dbusiness

49The disturbances continued night after night until they finally petered out on September 5At the Old Bailey Judge Salmon later handed down exemplary _____ of four years each on nine white youths who had gonenigger hunting”.

Adecisions Bstatements

Ctrials Dsentences

50While those dealt with by the courts were overwhelmingly white the large number of black people also arrested and the official _____ there had not been a racial motive ensured a legacy of black mistrust of the Metropolitan police that has never really been eradicated

Apersistence Bperseverance

Cinsistence Dinstance

Section B Error Correction10points

DirectionsThe following passage contains 9 errorsIn each case only one word is involvedYou should proofread the passage on the Answer Sheet and correct it in the following way

EXAMPLE

One nightquite lateI was still awake in the room I am shared with 1. am

my husbandI was lying on my right side and can hear a child crying 2. could

Getting upI went see if our son was all right 3. to

He was sleeping soundlybreathing deeply and gently 4.

The Zipper

Whatever did we do before the invention of the zipper

In 1893 the world's first zipper was produced in Chicago

Although the inventor claimed that it was a reliable fastening

for clothingthis was not the caseThe Chicago zipper sprang 51______

open without warningor jammed shutand it swiftly lost

popularityTwenty years ago a Swedish-born engineer called 52______

Sundback solved the problemHe attached tiny cups to the

backs of the interlocked teethand this meant that the teeth 53______

could be enmeshed more firmly and reliably

At first zippers were made of metalThey were heavyand

if they got stuck it was difficult to freeThen came nylon 54______

zippers which were lighter and easier to useand had smaller

teethThe fashion industry liked the new zippers far better

because they didnt distort the line of the garment or weighing 55______

down light fabricsThey were also easier for the machinists

to fit into the garment

Meanwhile a new fastening agent made its appearance at

the end of the twenty century: velcro. Velcro is another product 56______

made from nylonNylon is a very tough synthetic fibre first

developed in the 1930sand bearing a name to mind the wearer 57______

of the two places where it was developedNY for New York and

LON for LondonVelcro is made with very small nylon hooks on

one side of the fastening which caught tiny looped whiskers on the 58______

other side of the fasteningIt is strong and durable

Velcro is used on clothingluggages and footwearIt is quick 59______

and easy to fasten and unfastenand has taken a large part of

the zipper's share of the marketIt is also used in ways a zipper

cannot be used—for instance as an easily changed fastening on

plaster castsand to hold furnishing fabrics in a position 60______

Part III Situational Dialogues5 minutes10 points

DirectionsComplete the following dialogues by choosing the best answerThen mark the corresponding letter on the Answer Sheet with a single line through the centre

61RobHey Jillyou're looking great

JillThanksRob____________

RobWellyou did itHow

Jill I jog every morningand I go to aerobics every other day

AI bought this dress yesterdayReally smart

BYou are looking fine too

CI'm recovering my strength after the flu

DMy New Year's resolution was to get in shape

62BobHi JaneHow are you

Jane____________I didn't sleep a wink last nightThe people next door were making a lot of noise again till very late at night

AI'm feeling a bit out of sorts this morning

BFinethank youAnd you

CI slept like a log and didn't want to get out of bed

DIt seems a bit unusualyou know

63Ann AahHe's gorgeousLook at those biggolden pawsWhen did you get him

RogerYesterday____________

Ann OhrightWhat kind is she

RogerA Labrador

ASusan's got a more beautiful one

BWhat's up

CIt's a she actually

DIsn't it right

64TinaWowlook at all the things on sale____________

AndrewYeslookthis shirt is 50 off

TinaAnd look at these shoesThey are 30 off the normal price

AI'd like to buy a skirtBThere are some real bargains

CAre the prices reasonableDThese shoes are the same as mine

65WomanHave you finished the packaging

Man ____________

WomanGoodBecause the truck will be coming soonthis is a rush job

ADon't hurry m or I'll break the glass

BAlmostI just have to wrap the glass and put it into boxes

CNoI haven'tWhy didn't you help me with it

DYesI haveWhat else can I do for you

66Customs Officer ________________________

MrsJohn son Nonothing at all

Customs Officer No perfumealcohol or cigarettes

MrsJohn son WellI have 200 cigarettesthat's all

ADo you have anything in the bagma'am

BDo you have anything to declarema'am

CDo you want to buy somethingma'am

DIs there anything I can do for youma'am

67LindaHelloI'd like to send this packageplease

Clerk____________________________________

LindaFirst classHow long will that take

ClerkAbout three days

AHow would you like to send it BWhich class are you in

CWhere do you want to send it toDWhich class is it in

68AssistantCan I help you

Colin Yesit's about this sports shirtI washed it the other dayThe colour ran and it shrank

AssistantOh dearI see________________________

Colin I'm afraid not

AssistantI'm sorrybut I'm not allowed to change anything without a receipt

ADid you buy it here

BWould you want to change it

CDo you have the receipt

DCould you tell me who sold it to you

69JamesCould I have my billpleaseCan I pay by credit card or eurocheque

Receptionist____________

JamesI'll pay by credit cardthen

ReceptionistThat's fineI hope you enjoyed your stay here

AHere's your billBSorrywe don't take credit card

CYou can pay by eurochequeDYeswe take both

70HusbandWhen is our anniversary

Wife________________________

HusbandNoit's just that I bought these flowers for you and I was hoping today was the day

AHmm ...I can't remember eitherWhy

BHeyare these flowers for me

CWho caresDo you want to give me a surprise

DAre you jokingHave you really forgotten again

Part IV Reading Comprehension25 minutes40 points

Section A Multiple Choice10 points

DirectionsThere is one reading passage in this partThe passage is followed by 5 questions or unfinished statementsFor each of them there are four choices marked ABC and DYou should decide on the best choice and mark the corresponding letter on the Answer Sheet with a single line through the centre

Questions 71 to 75 are based on the following passage

Taking a peep at what's going on in your head

CARL Filer18a star salesman at a BQ hardware store in the UKwas called up for promotion within one week of starting workButinstead of being made supervisorhe was sacked—after his employers saw the results of his psychometric test

You might think that anyone who answers that hestrongly disagrees he is an over-achiever is asking for troublebut Mr Filer already thought he had proved himself more than capable

This yearnearly half of UK firms—46 percent—will use psychometric tests to select traineescompared with just 17 percent in 2000according to a report for GTIa publisher of graduate career guides

These testswhich rate candidates ability and gauge their personalityhave been used in the UK since the 1980sBut assorted studies have shown most people—graduates in particular—are wholly cynical about the idea of their personality beingmeasured

People tend to see them as either too silly or too clever says Clive Fletcherprofessor of occupational psychology at University of London.“But all the evidence indicates the tests do have some value

The first personality test as we know itwas developed by the American army in 1917 to filter out weak recruits

But it was not until the 1980s that the tests became popular in BritainWith a rising number of graduates going for a decreasing number of jobsorganizations began to see psychometric testing as a cheapreliable alternative to the expensivetime-consuming interview

But today the tests are becoming alarmingly sophisticated and are edging towards probing thedark sidepathology and personality disordersIncreasinglytests are being used to try to detect promising young graduates who maylater in lifefly off the railsgo crazy);or to stop psychopathshaving mental disordergetting recruited

In the futureinterviewees could even be given a mouth swab to reveal the genetic and biological markers of personality.“We are heading for the era of genetic screening,”warns Carolyn Jonesof the Institute for Employment Rights.“I think these tests are very flawed

And there are other problems with the testsFor startersit is possible to fake it—even the test producers agree on thisBut they have made it as hard as possibleFor example look at whether you agree or disagree with the following two statements:“New ideas come easily to me andI find generating new concepts difficult How long did it take you to realize they both could mean the same thing

The main argumenthoweveris that the tests are invalid and cannot quantifyput a numerical value onsomething as changeable as personality

The golden rule is thenthat a psychometric test should never be used as the sole basis of selectionbut should always be followed by interviews

71Most people's attitude towards the psychometric test is ______

Acontemptuous Bfavorable Ctolerant Dconfounded

72Which of the following is one of the reasons why psychometric testing wins an advantage over interviews

AIt doesn't cost any money BIt requires no equipment

CIt is time-saving DIt can be done within seconds

73Which of the following statements is the author's idea

APsychometric tests are defective

BPsychometric tests should not be the only way to recruit promising young graduates

CPsychometric tests are invalid and cannot quantify something changeable as personality

DPsychometric tests are golden rules

74The test producers make the tests very complicated to ______

Aavoid cheating Bimprove genetic screening

Cfind out the best ideas Dgenerate new concepts

75Which of the following is not true according to the passage

AThe American army developed the first personality test to screen out weak recruits

BIn the futureinterviewers could give a mouth swab to reveal interviewees symptoms

CThere are possibilities for starters to cheat in the psychometric tests

DInterviews still play an important role in evaluating interviewees

Section B Short Answer Questions30 points

DirectionsIn this part there are 3 passages with 15 questions or incomplete statementsRead the passages carefullyThen answer the questions in the fewest possible wordsnot exceeding 10 words).Remember to rewrite the answers on the Answer Sheet

Questions 76 to 80 are based on the following passage

The 8 Steps of Social Invention

1Get ready to play

Like other types of creativitysocial inventiveness flourishes when you begin thinking outside conventional boundariesCharlie Girscha StPaulMinnesota-based creativity consultantsuggests that you start by playing with obviously absurd explanations for everyday events.“If traffic is slowyou'll be tem pted to say,‘HmmMust be an accident up ahead.’ Insteadtry saying,‘Must be a family of turtles crossing the highwayorI expect there's some kind of alien abduction going on.’You'll be amazed how soon you will be looking at familiar problems in new ways.”Girsch's bookFanning the Creative SpiritCreativity Central1999has scores of other exercises for limbering up the inventive part of your brain

2Generate a zillion far-fetched ideas

Concerned about the homeless in your neighborhoodImagine a Homeless Parliamenta Homeless Circushomeless families forming an orchestraa homeless museum ...and on and onGenerate like mad with no regard for feasibility in orderas social invention pioneer Nicholas Albery advisestoovercome e worthy-but-dull ideas.”Eventually the two or three best ideas will begin to stand out

3Take your wildest idea and bring it down to earth

How about that Homeless CircusCould it turn into a forum for homeless people to display their creative talentsA performance series about homelessnessA neighborhood carnival with the homeless as guests of honorYour flakiest idea may have a germ of brilliance that actually makes it more attractiveand thus more feasibleand fundable),than its worthy-but-dull cousins

4Look for in venations that solve more than one problem

The Slow Food Movementborn in Italyboosts local farmers and regional cuisine traditions and restaurateurs and the same time that itfeedsour hunger for authentic tastes healthy eatingand a more leisurelysaner style of life

5Accentuate the positive

A very common question that I get when I work with people in communities isWhy doesn't anybody care about our problems?’”notes M chael Pattersona social inventor and activist in Massachusetts.“What a worthless question.‘Why’?questions are for philosophersAskHow’?andWhat’?questions—they are a lot more practical.”For instance Patterson asks,“What would you do if you knew you couldn't fail?”

6Give it a rest

Walk away from your favorite idea for a whileforget about itlet it sleepWith your conscious mind out of the wayyour subconscious gets to fiddle with the concept for a while and you just might have an unexpected insight or breakthrough

7Practiceyes andin stead ofyes but”.

No matter how tempted you are to sayYesbut this will be hard because,”orYesbut a million other people are doing this,”shift the conjunction toandand see what sort of positive refinement or change emerges.“Yesand we could concentrate on immigrants.”“Yesand we can make it open to all ages.”

8Get your idea into the world

This is the tough partYou might seek out the help of activists who will take a shine to your ideasOr become an organizer yourselfPaul Glovea New York social inventorcoun-sels:“If you have an idea you believe inwrite a pamphlet with your phone number on it and post it in Laundromats and bookstoresIf three people call youhave lunch with them and call yourselves an organizationIf five people callmeet with them and issue a press release.” Prestoyou're launched

76To generate far-fetched ideas helps to ______

77Michael Patterson wants us to come up withHow”?andWhat”?instead ofWhy”?questions because he considers they are more practical than ______

78The purpose to practiceyes andinstead ofyes butis to make yourself more ______

79According to the articlewhen one has difficulty developing his favorite ideahe should ______

80One should not only generate far-fetched ideas but also ______ because the latter step is the nearest to reality

Questions 81 to 85 are based on the following passage

Thin Slice of TV Has Big Market

It is too early to write an obituary for bulky picture tubeswhich will remain the most affordable TV sets for years to come

Butanalysts and industry executives insist that thin screens already have started to become the dominant format for TV sets in the digital era

Sharp price cuts have brought plasma sets and other thinflat televisions out of high-end electronic boutiques and into thousands of mass-market outlets such as Coscoa wholesale buying club in the USbest known for offering members bulk items and big discounts

The least expensive plasma sets still cost a hefty US 3000or more yet sales are growing so rapidly that many manufacturers are racing to boost production

That increasecombined with expanding production capacity and improved technology could push the price of plasma sets down by one-third next yearaccording to analyst Richard Doherty of Envisioneering Groupa US research firm

But manufacturers are not just competing with each otherthey are also trying to fend off challenges from competing thin-screen technologiessuch as liquid crystal displaysLCD).

The demand for thin screens is fuelled in part by the advent of DVDs and digital TV broadcastswhich offer more detailed pictures and more lifelike colors than conventional analog TV signals

To see the differenceconsumers need a set that can pack more information onto the screen than their current TVs can

This sharpness is most vivid on screens that are 40inches diagonal or largerAt that sizehowevertraditional direct view and projection TVs are so bulky that many consumers have trouble finding a place for them at home

Hence the interest in thin screens—models slender and light enough to hang on a wall

The glass panels at the heart of plasma and LCD sets come mainly from about a dozen companies with factories in JapanSouth Korea andincreasinglyChinaAbout 800000 plasma panels will be shipped this year around the worldanalysts say

That is a tiny amount compared with the overall market for TVswhich was about 140 million sets last yearButindustry experts said 2003would be abreakout year or plasma because shipments should double

Helping drive the growth are new or expanded manufacturing facilitiesFor example Japanese electronics giant NEC last year doubled the capacity of its Japanese factory—reaching 300000to 400000 plasma panelsAnd it plans to double it again in 2003officials said

As competition has heated up during the last four yearsprices have fallen more than 50 percentAccording toNPD Tec world”,the average price of a plasma display sold in the US dropped from US 12700in January 1999 to US 6100in October 2002

The best markets for plasma screens have been in Asiaand about half of the sets have gone to businesses instead of homes

LCD TVs carry a premium price—they can be 10 times as expensive as a comparable tube-driven television—that knocks them out of most buyers budgets

But LCD panels are quickly taking over the market for computer monitorsand the tens of millions of panels being produced for that segment will help push down prices for LCD TVsanalysts predicted

Sharp Electronicsfor oneis betting heavily on LCDsIts chairmanToshiaki Urushisako has predicted that Sharp will switch completely from conventional tube sets to LCD TVs in Japan by 2005

Flat-panel refers to wafer-thin3 inches or lessTVswhereas flat-screen may actually describe traditional cathode-ray-tube setsCRTswhose glass front lacks the distorting curve that TVs have had for 50 years

Be aware of two thingsOneflat-panel technology may not be high-definition TVfor eventual HDTV receptionsome of these sets will require a separate HD tunerTwosome flat-panel TVs are just the panel and lack speakers and sometimes a built-in tuner

Price rangeUS 7002000

LCD vs plasma

In generalLCD technology is used for smaller screens because of the enormous number of transistors needed to turn the glasslike liquid crystals into color imagesThe larger the displaythe more transistorsthe more chance of failed connections

A plasma screen is found in TV sets larger than 20 inchesColor is comparable to an LCD'sLCDs do not deteriorate over timewhile a plasma display averages 30000 hoursa traditional TV screen can go for 20000),after which it fades over a period of yearsEarlier problems with the quality of plasma's contrast have been addressedand current screens are cleaner and better defined

Price rangeUS 6002800

HDTV

Simply puthigh-definition TV is 10 times as sharp as traditional TVand the sound is digitallike CD soundnot FM which is what traditional TV providesHD technology achieves its visual clarity with more immage lines on the screenWhere analog TVs have 480 horizontal linesHDTV has 720 or 1080linesBe aware m any HDTV sets being sold now are in fact only HDTV monitors offering a crisp picture To receive genuine high-definition television signalsowners must buy a separate HDTV receiver

Price rangeUS 10006000

LOS ANGELEST IMES

81According to the articleTV sets with _____will still be the most popular in the coming years

82The factors that stimulate the thin screens to be more and more popular include __________and_____

83The rapidly expanding market for LCD panels and their large-scale production will help lower _____

84When a wealthy customer wants to buy a very large TVhe should select _____and _____according to the passage

85We can infer from the passage that among all kinds of TV sets _____is of the best quality

Questions 86 to 90 are based on the following passage

Dell Does Domination

Over the years I've spent a fair amount of time hanging out with Michael Delland what I noticed during my latest visit with him in Austin is how things have changedYeshe is still unflappableAnd yeshe greets me in his new glossy offices with the same Stamford Wife-like grin he has always hadBut he appears thinner nowas if he's lost baby fatWhile he's still slow-movingas if he's conserving energyhe now cuts to the quick in conversationAnd when he zeroes in on the point he wants to makewhen he reiterates why Dell Computer is in a better position than any other PC maker in the worldyou realize that the 36-year-old has lost what was once one of his greatest advantagesno one underestimate ates him anymore

InsteadMichael Dell looms over the PC landscape like a giantcasting a shadow over all his unfortunate com petitorsThis is a terrible time in a difficult businessPC sales were down for the first time last yearDell's sales will be downtooalso for the first timeYet even with thateven with recession even with the threat of a Hewlett-Packard Com paq Goliaththis is the only PC maker you can count on to grow and grow and growAlmost single-handedlyDell is forcing this industry to consolidateCould this meangame overin the PC biz?“Game over?”he looks back at me incredulously.“No wayWe only have 14 global market share.”

The Dellites may not admit togame overaspirationsbut clearly they are thinking of a kind of domination never seen before among PC makers.“We think 40market share is possible,”says Dell's No2Kevin RollinsThat's a remarkable goalwhat's more remarkable is that it really is attainableDon't look for Dell to hit that kind of number anytime soonRatherthe company's growth will come from grinding out gains on several existing frontswhile shrewdly expanding into new target markets

The reason is simplethere's no better way to makeselland deliver PCs than the way Dell does itand nobody executes that model better than DellBy now most business people can recite the basic tenets of Dell's direct-sales modelDell machines are made to order and delivered directly to the customerThere is no middlemanThe customer gets the exact machine he wants cheaper than he can get it from the competitionThe company gets paid by the customer weeks before it pays suppliersGiven all thatthe company that famously started in Austin out of a University of Texas dorm room now dominates the northern side of this city the way giant steelworks once lorded over old mill townsDell has some 24 facilities in and near Austin and employs more than 18000 local workersDell did over 30billion in sales in 2000ranking 48th on the FORTUNE 500ahead of names like Walt Disneyand Du PontMichael is the richest man under 40 in the worldworth 16 billion

Two facts show how well the Dell model is workingeven in tough timesDell is on track to earn over 1.7 billion in 2001taking almost every single dollar of profit among makers of Windows-based PCsAnd Dell is gaining market shareThat's not true for any other major PC maker

Quite the contraryThe others are going splat for the same reason that Dell is succeedingcommoditizationThe desktop PC has become a commodityThat's great for consumerswho get standardizedeasy-to-usecheap PCs

Commoditization has been going on in the industry for yearsDellas master of the direct modelspent most of the 1990s operating in techno-NirvanaThe PC market was growing by 15-plus per yearFor its quarter ended January 2000Dell did a record 68 billion in salesup 31 from the previous year's quarterIn a sign of things to comesales growth slowed later in 2000Then the growth disappeared in 2001

The economic slowdown was bad news for everyonebut Michael Dell and Kevin Rollinswho is increasingly his equal partner in running this businessmade sure it was terrible news for Dell's com petitorsIn late 2000they decided to slash prices.“It was advantageous for usactuallybecause in periods of slow demand component prices dropand unlike our competitionwe can pass those savings on immediately to customers,”explains Rollinsa fine violinist who grew up in a hard-charging Utah family—his father was an engineering professor at Brigham Young—and came to Dell from the Bain consulting firm Dell could make more money selling more computers at lower prices than it could selling fewer computers at higher pricesThe low prices wreaked havoc on competitorsCompaqHPand Gateway all lost market share for the 12 months that ended Sept302001while Dell's share of the USmarket climbed 31%.

86_____was once one of Dell's greatest advantages

87The passage attributes the success of Dell to_____

88Why did Dell's share of the U.Smarket climb 31 in 2001 while the economy slowed down

89Dell hopes to increase its global market share by _____according to the passage

90If one wants to buy a Dell computerwho do you think he is supposed to pay

Part V Word Guessing and IQ Test5 minutes10 points

Section A Word Guessing5 points

91In the western countriesa person who serves the guests in a restaurant expects a gratuity in appreciation for good service

Agood remark Bletter of thanks Ctip Dpromotion

92In Greek mythologythe hero usually finds himself in a terrible predicament in which he has to face a fate which has been foretold by the Oracle at Delphi

Astrange phenomenon Bunpleasant situation

Cunusual circumstance Dhuge monster

93A first-class business establishment will fully recompense its customers if there is ever any dissatisfaction with the service or product

Acompensate Bunderstand Cinform Daward

94Our family was truly a matriarchybecause my grandmother had to raise her children and provide for them all by herselfmy grandfather had died when his sons and daughter were quite young

Afamily governed by women Bfamily without the father

Cfamily depending on the mother Dfamily with many young children

95I'd love to help you look for all those things but I'm a bit pushed for time todayWhy don't we split up and meet back in a couple of hours for a quick cuppa and then I can have a look at what you've bought

Aput it aside Bgo separately

Csave the time Ddo it one by one

Section B IQ Test5 points

96What number should replace the question mark

A10 B9 C8 D7

97If someone could take a sheet of paper 01mm thicktear it in halfput the pieces on top of each othertear these in halfthen repeat the process until the paper had been torn 25 timeswhat would be the final height of the pile of paper

AAs thick as a book BAs tall as a man

CAs high as a house DAs high as a mountain

98If the pyramid were flattened out would it look like ABC or D

99Only one of the sets of six letter below can be rearranged into a six-letter word in the English languageCan you find that set

ALO RIDM BETNMIU CTUBLID DMIRCEL

100These are millennium celebrations taking place in 4 different placesWhich one occurred first

A. PARIS B. SYDNEY

C. NEW YORK D. GREAT WALL OF CHINA

Part VI Translation10 minutes10 points

DirectionsTranslate the underlined sentences of the following passage into Chinese on the Answer Sheet

TASTE A WORLD OF DIFFERENCE

Wherever you are in the worldit seemsthe billboards and supermarket shelves are saturated with the same brand namesThere are few places left on the planet where you can't buy a Diet Cokea packet of Kellogg's cornflakesor Oxo cubes

But just try tasting them.(101The packaging and the product may look identicalbut the flavour maybe far from familiarEven the most famous brand names are specially for-mulated to appeal to individual national palatesHeinzfor instanceinsists that its tomato ketchup isthe same recipe worldwide”,but admits thatthere maybe very subtle variations in the spicing”.

Even the ultimate global brandCoca-Colaaccepts that not all its products are what they appear to be.“We go to great lengths to ensure that Coca-Cola is the same wherever you drink it,”says a spokesman.“But Diet Coke may change slightly from country to countrybecause we use different sweeteners in different places.”

Americans prefer many productsparticularly chocolateto be far more sugary than Europeans do.(102The French perceive strawberry flavour in a different way from the Britishso the artificial flavouring in France will not have aproperstrawberry taste to BritonsPreferences for saltiness and color differ as welland most Japanese consumers dislike the taste of milk

The French in general prefer strongerfuller-tasting coffee than the BritishThere is a higher cream content in a Magnum in Italy than in Britainbecause the Italians expect ice-cream to have a much richer tasteTea sold in Europe tends to be weaker than the strong tea preferred by Britons.(103Taste preferences can vary even within countriesat least one well-known soft drink is made according to a sweeter recipe in the southern states of the United States than that on sale in the north

Local water and soil will affect the taste of home-grown natural ingredientsSome key constituents may not be availablemeaning that alternatives must be foundSometimesespecially in developing countriespremium grade componentssuch as high-grade flourare replaced with lower-quality equivalents

104Extremes in climate will require different additives and preservatives to be used Chocolate calls for an alternative recipe in hot countries if it is to maintain its texture and taste

And multinationals must conform to national laws and regulations on additives flavoringscoloringsand artificial low-calorie sweetenersMars and Snickers bars taste slightly different in Australiasince laws there state that vegetable fat must not be used in chocolate

Local religious sensibilities must also be observed.(105Even McDonald'swhich proclaims the homogeneity of its Big Macs(巨无霸)around the globehas to serve lamb rather than beef burgers in Indiabecause the sacred status of cows means that the majority of its Indian customers can't eat beef

Part VII Writing30 minutes20 points

DirectionsThe chart below shows the amount of leisure time enjoyed by men and women of different employment statusWrite a report for a university lecturer describing the information shown belowYou should write at least 150 words

Leisure time in a typical weekby sex and employment status199899

2003年全国大学生英语竞赛初赛赛卷答案

Part I Listening Comprehension (30 )

Section A Dialogues (10 , 每题1)

1. C. 2. C 3. C 4. B 5. D 6. C 7. A 8. A 9. B 10. B

Section B News Items (10 , 每题1)

11. The UN Security Council’s resolution.

12. $100 million.

13. About a deadly shooting.

14. 14.

15. A way to reduce early births among women at the risk of premature delivery.

16. The Bush administration.

17. A fire in a night club.

18. To seek UN approval for war.

19. Murder and injury.

20. A gunshot.

Section C Compound Dictation (10 分,21—28题,每题0.5分;2930题,每题3)

21. purchasing 22. firms 23. engage 24. factors 25. concerned

26. climate 27. introducing 28. characters

29. This critical problem of proper translating is only one of many cultural differences facing American corporations overseas

30. Because of their cultural heritage, businesspersons in each country conduct their activities differently.

Part II Vocabulary and Structure (30 )

Section A Multiple Choice (20 , 每题1)

31. B 32. D 33. A 34. C 35. C 36. A 37. D 38. B 39. B 40. A

41. D 42. C 43. B 44. C 45. B 46. B 47. A 48. D 49. D 50. C

Section B Error Correction (10 , 每题1)

51. 52. ago later 53. interlocked interlocking 54. free∧→ them

55. weighing weigh 56. twenty twentieth 57. mind remind 58. caught catch

59. luggages luggage 60. a 删除a

Part III Situational Dialogues (10 , 每题1)

61. D 62. A 63. C 64. B 65. B 66. B 67. A 68. C 69. D 70. D

Part IV Reading Comprehension ( 40 )

Section A Multiple Choice (10, 每题2)

71. A 72. C 73. B 74. A 75. B

Section B Short Answer Questions (30, 每题2)

76. increase your social inventiveness

77. theoretical/philosophical

78. determined

79. give it a rest

80. get them into the world / make them known

81. bulky picture tubes

82. price cuts, convenience, the demands for high definition

83. prices for LCD TVs

84. LCD, HDTV

85. HDTV

86. That he was once underestimated / That someone once underestimated him

87. The direct sales model / commoditization

88. They slashed the prices. / They lowered the prices.

89. 26%

90. The Dell company.

Part V Word Guessing and IQ Test (10)

Section A Word Guessing (5 分,每题1)

91. C 92. B 93. A 94. A 95. B

Section B IQ Test (5 分,每题1)

96. C 97. D 98. A 99. B 100. B

Part VI Translation (10 points)

101. 包装和产品也许看起来是完全一样的,但是味道也许跟所熟悉的相差甚远。

102. 法国人对草莓口味的感觉与英国人不同,所以法国产的草莓人工调味剂不合英国人的胃口。

103. 甚至在一国之内,口味偏好也会有极大差别:至少一种指明软饮料,在美国南部地区是用比在北方销售的饮料甜味更重的配方制成的。

104. 气候的巨大差别要求使用不同的添加剂和防腐剂。

105. 即使宣称在全球销售同一种类的巨无霸的麦当劳也不得不在印度提供羊肉汉堡而不是牛肉汉堡,原因是牛的神圣地位意味着大多数印度顾客不能食用牛肉。

Part VII Writing (20)

(参考范文)

The chart shows the number of hours of leisure enjoyed by men and women in a typical week in 1998-99, according to gender and employment status.

Among those employed full-time, men on average had forty-seven hours of leisure, whereas women had approximately thirty-seven hours. There were no figures given for male part-time workers, but female part-timers had thirty-nine hours of leisure time, only slightly more than women in full-time employment, perhaps reflecting their work in the home.

In the unemployed and retired categories, leisure time showed an increase for both sexes, as might have been expected. Here too, men enjoyed more leisure time---over eighty hours, compared with less than seventy hours for women, perhaps once again reflecting the fact women spend more time working in the home than men.

Lastly, housewives enjoyed approximately fifty-four hours of leisure, on average. There were no figures given for househusbands! Overall, the chart demonstrates that in the categories for which statistics on male leisure time were available, men enjoyed at least ten hours of extra time.

2004年全国大学生英语竞赛初赛试题

2004 National English Contest for College Students

(Preliminary)

Part I Listening Comprehension (30 minutes, 30 points)

Section A Dialogues (10 points)

Directions: In this section, you will hear 10 short dialogues. At the end of each dialogue, a question will be asked about what was said. Both the dialogue and the question will be read only once. After each question ,there will be a pause. During the pause, you must read the four choices marked A, B, C and D, and decide which is the best answer. Then mark the corresponding letter on the Answer Sheet with a single line through the centre.

1. A. In San Francisco. B. At an airport.

C. At a travel agency. D. In a post office.

2. A. The woman is going out to lunch.

B. The woman wants to eat some chocolate.

C. The woman will go to a convenience store.

D. The woman will be back in 30 minutes.

3. A. By car. B. By plane.

C. By train. D. By ferry.

4. A. She had lost her job.

B. She didn’t know the mayor.

C. She was mistaken.

D. The man misunderstood her.

5. A. He needs some tomato juice.

B. His shirt is stained.

C. He needs his shirt by tomorrow.

D. His shirt is missing.

6. A. To a meeting. B. To the office.

C. To a restaurant. D. To a bowling class.

7. A. Give the woman some medicine.

B. Find out more about the woman’s injury.

C. Test the strength of the woman’s shoulder.

D. Go skiing with the woman.

8. A. Excited.

B. Thankful.

C. Somewhat disappointed.

D. Somewhat bothered.

9. A. When her family celebration is over.

B. After the man graduates from school.

C. After they have some pictures taken together.

D. When she has bought her cap and gown.

10. A. By continuous assessment.

B. By giving a per cent.

C. By giving grade.

D. By means of exams.

Section B News Items (10 points)

Directions: In this section, you will hear 10 short pieces of news from BBC or VOA. After each news item and question,there will be a pause. During the pause, you must read the three choices marked A, B and C, and decide which is the best answer. Then mark the corresponding letter on the Answer Sheet with a single line through the centre.

11. A. Under the age of four.

B. Under the age of five.

C. Under the age of six.

12. A. Beijing’s successful bid for the 2008 Olympic Games.

B. The Organizing Committee of the 2008 Olympic Games.

C. The large market of the Olymic brand.

13. A. To keep the code for its Windows operating system a secret.

B. To design some new computer software.

C. To persuade more PC users to adopt the Windows operating system.

14. A. One. B. Ten. C. Thirty.

15. A. No. B. Yes. C. Not mentioned.

16. A. More than 500 dollars.

B. A little more than three dollars.

C. Less than three dollars.

17. A. Because the Iraqi economy has gradually risen after the war.

B. Because Iraqi people trust the new dinar more.

C. Both A and B.

18. A. Five. B. Six. C. Seven.

19. A. Low fruit and vegetable intake.

B. Smoking and little exercise.

C. Unhealthy diet.

20. A. The euro has risen in value.

B. The US dollar has risen in value.

C. German economy has slided into recession.

Section C Passages (10 points)

Directions:In this section, you will hear 2 passages. At the end of each passage, you will hear 5 questions. After you hear a question, you must choose the best answer from the four choices marked A, B, C and D. Then mark the corresponding letter on the Answer Sheet with a single line through the centre.

Passage One

21. A. Jazz. B. Indian. C. Country. D. Pop.

22. A. Their hairstyles. B. Their humor.

C. Their clothing. D. All of the above.

23. A. America. B. England.

C. Italy. D. Canada.

24. A. The Beatles were formed in England.

B. The Beatles had a successful movie career.

C. The Beatles are regarded as one of the finest jazz groups.

D. The Beatles first recorded music in 1962.

25. A. Seventeen. B. Twenty-eight.

C. Twenty-two. D. Twelve.

Passage Two

26. A. Because he regarded the bear as his friend.

B. Because the bear was beautiful.

C. Because he considered it poor sportsmanship to shoot a tied-up animal.

D. Because bears are not dangerous animals.

27. A. Because Teddy is the nickname for Theodore Roosevelt.

B. Because it was then the usual practice to do so.

C. Because it was the first time to name toy bears Teddy Bears.

D. Because the toy bear was made to look a bit like the President.

28. A. People collect Teddy Bears.

B. Teddy Bears can be found in museums.

C. The first Teddy Bear was made by Mr. Mitchtom’s wife.

D. President Roosevelt shot the black bear in 1902.

29. A. Seven. B. Six. C. Two. D. Five.

30. A. He moved to Florida.

B. He became President.

C. He drew cartoons.

D. He started a toy company.

Part II Vocabulary and Structure (10 minutes, 20 points)

Section A Multiple Choice (10 points)

Directions:There are 7 incomplete sentences and 3 incomplete dialogues in this section. For each blank there are four choices marked A, B, C and D. Choose the one that best completes the sentences and dialogues. Then mark the corresponding letter on the Answer Sheet with a single line through the centre.

31. Never________the power of your actions. With one small gesture you can change a person’s life.

A. underestimate B. overvalue

C. misuse D. dismiss

32. Scientists have warned that penguins in the Antarctic could be very________to changes in climate and could be threatened by any long-term temperature shifts.

A. superstitious B. acceptable

C. suspicious D. susceptible

33. Since settling in Scotland I ________ golf as a hobby.

A. have taken up B. took up

C. have taken in D. took in

34. She often thinks that her six years in Italy were wasted, ________she________ that time learning more Italian.

A. but that; might have taken

B. for that; should have found

C. in that; could have spent

D. with that; would have used

35. He constantly________his proposal that________of the budget surplus be used to offer a voluntary prescription drug benefit to seniors.

A. views; many B. reiterates; a part

C. complains; a great amount D. thinks; lots

36. ________Alan’s amazement, the passport office was closed when he arrived.

A. With B. For C. To D. Of

37. I was asked the other day whether high and low pressure systems were________the central pressure.

A. maintained to B. determined by

C. generated within D. preserved to

38. Bob: What are you reading, Frank?

Tom: It’s this week’s New Scientist, why?

Bob: I was just wondering—________, but I’ve never actually read it myself. Is it aimed at real scientists or can ordinary people like me understand it?

A. it’s for anyone really B. where I can buy it

C. it seems very expensive D. it looks interesting

39. Girl: Hi Paul—looking forward to your holiday?

Boy: Oh, yeah—it’s going to be great. Though I’m a bit worried that I’ve packed the wrong clothes. I don’t think the weather’s going to be as good as I hoped.

Girl:________

Boy: That’s right—my first flight.

A. Everything will be OK, isn’t it?

B. You’re flying on Saturday, aren’t you?

C. It’s far from here, as everybody knows.

D. That’s a good idea, anyway.

40. John: What plastic products do you have in mind that are easy to recycle?

Tom: Shampoo bottles, detergent bottles, medicine bottles, food containers, etc. They are all easily collectable and reusable.

John: ________, but actually I think you are missing the point of recycling. It doesn’t just mean using old bottles again and again for the same purpose. What it means these days is melting the plastics down and building them up again into some completely new product.

A. Not too bad B. Something is wrong

C. You’re right there D. It’s a new idea

Section B Cloze-Test (10 points)

Directions: There are 10 blanks in the passage. For each blank there are four choices marked A, B, C and D. Choose the one that best completes the sentence. Then mark the corresponding letter on the Answer Sheet with a single line through the centre.

Ask most people for their list of Top Ten fears, and you’ll be sure to find being burgled fairly high on the list. An informal survey I carried out among friends at a party last week (41)________that eight of them had their homes broken into more than twice, and two had been burgled five times.To put the record (42)________, none of my friends owns valuable paintings or a sideboard full of family silverware. Three of them are students, in fact. The most (43)________burglary, it seems, involves the theft of easily transportable items—the television, the video, even food from the freezer. This may have something to do with the fact that the average burglar is(44)________his (or her) late teens, and probably wouldn’t know what to do with a Picasso, (45)________selling a Walkman or a vacuum cleaner is a much easier matter. They are perhaps not so much (46)________criminals as hard-up young people who need a few pounds and some excitement. (47)________that this makes having your house turned upside down and your favourite things stolen any easier to accept. In most cases, the police have no luck (48)________any of the stolen goods. Unless there is any (49)________evidence, they are probably unable to do anything at all. And alarms or special locks don’t seem to help either. The only advice my friends could (50)________up with was “Never live on the ground floor” and “Keep two or three very fierce dogs”.

41. A. released B. revealed C. reclaimed D. redeemed

42. A. straight B. clear C. apparent D. correct

43. A. typical B. abnormal C. hazardous D. vicious

44. A. near B. in C. beyond D. out of

45. A. whereas B. whenever C. however D. once

46. A. serious B. professional C. efficient D. perfect

47. A. Given B. Even C. Not D. Despite

48. A. seizing B. withdrawing C. seeking D. recovering

49. A. distinguishable B. obscure C. outstanding D. definite

50. A. come B. catch C. keep D. put

Part III Word Guessing and IQ Test (5 minutes, 10 points)

Section A Word Guessing (5 points)

51. Social capital has become a mantra for politicians and policy makers: they see it as a bulwark against society’s ills and a means of multiplying the effects of financial investment in social projects.

A. measurement B. defense

C. treatment D. complaint

52. Her desire for anonymity soon became apparent when she refused to answer questions about her identity.

A. recognition B. concealment

C. vanity D. success

53. “Gentlemen,” replied Candide, with a most engaging modesty, “you do me much honor, but upon my word I have no money.”

A. you help me a lot B. you are great

C. it’s very generous of you to say so

D. that’s a great honor for me

54. When it comes to listening to the opinions of members of your school community, do you think you are already “all ears?”

A. in full strength

B. bearing ideas in mind

C. ready to listen attentively

D. having enough preparation

55. That extremely indolent student will clean out his desk when pigs fly or I am much mistaken as to his character.

A. soon B. never C. sometime D. often

Section B IQ Test (5 points)

56. Sally had a third again as many as David, who had a third as many again as Francis. Altogether they had 111. How many did David have?

A. 27 B. 32 C. 36 D. 48

57. What letter should replace the question mark?

A. T B. S C. I D. N

58. BONA FIDE is to genuine as DE FACTO is to________.

A. together B. actual C. reason D. assumed

59. How many revolutions must the largest cog make in order to bring the cogs back to their original positions?

A. 56 B. 48 C. 36 D. 12

60. The diagram shows a small village church. There is a door in the west end, seen in the diagram. There is a tower at the east end of the church with a window set in its east wall. This wall is hidden in the diagram.There is also a door in the tower. Which of these is most likely to be the view of the eastern end of the church?

Part IV Reading Comprehension (25 minutes,30 points)

Directions:In this part there are 5 passages with 30 questions or incomplete statements. Read the passages carefully. Then answer the questions in the fewest possible words(not exceeding 10 words). Remember to rewrite the answers on the Answer Sheet.

Questions 61 to 66 are based on the following passage:

Centuries ago, man discovered that removing moisture from food helps to preserve it, and that the easiest way to do this is to expose the food to sun and wind.

Fruit is sun-dried in Asia Minor, Greece, Spain and other Mediterranean countries, and also in California, South Africa and Australia. The methods used vary, but in general, the fruit is spread out on trays in drying yards in the hot sun. In order to prevent darkening, pears, peaches and apricots are exposed to the fumes of burning sulphur before drying. Plums, for making prunes, and certain varieties of grapes for making raisins and currants, are dipped in an alkaline solution in order to crack the skins of the fruit slightly and remove their wax coating, so increasing the rate ofdrying.

Nowadays most foods are dried mechanically. The conventional method of such dehydration is to put food in chambers through which hot air is blown at temperatures of about 110 at entry to about 43 at exit. This is the usual method for drying such things as vegetables, minced meat, and fish.

Liquids such as milk, coffee, tea, soups and eggs may be dried by pouring them over a heated horizontal steel cylinder or by spraying them into a chamber through which a current of hot air passes. In the first case, the dried material is scraped off the roller as a thin film which is then broken up into small, though still relatively coarse flakes. In the second process it falls to the bottom of the chamber as a fine powder. Where recognizable pieces of meat and vegetables are required, as in soup, the ingredients are dried separately and then mixed.

Dried foods take up less room and weigh less than the same food packed in cans or frozen, and they do not need to be stored in special conditions. For these reasons they are invaluable to climbers, explorers and soldiers in battle, who have little storage space. They are also popular with housewives because it takes so little time to cook them. Usually it is just a case of replacing the dried-out moisture with boiling water.

Questions:

61. Fruit is sun-dried generally on________.

62. Why are sulphur fumes used before drying some fruits?

63. Where are vegetables commonly dried nowadays?

64. If soup requires recognizable pieces of meat, they are________.

65. Dried foods are often used by________, ________and________.

66. Why do housewives like dried foods?

Questions 67 to 72 are based on the following passage:

Hollywood writers honor Coppola, “Splendor”

Sunday, February 22, 2004 Posted: 9:56 AM EST (14:56 GMT)

LOS ANGELES, California (Reuters) —Hollywood’s screenwriters Saturday snubbed the final installment of the highly acclaimed Lord of the Rings trilogy and instead awarded a key prize to a low-budget film based on a comic book writer.

American Splendor, which revolves around the travails of comics connoisseur Harvey Pekar, won the Writers Guild of America Award for best adapted screenplay, while writer / director Sofia Coppola’s Lost in Translation, about a pair of mismatched Americans languishing in Tokyo, nabbed the trophy for original screenplay.

The American Splendor screenplay was written by the film’s directors, Robert Springer and Shari Springer Berman, who were not present at the awards. The Writers Guild of America Awards were handed out simultaneously in Los Angeles and New York.

Coppola said she was excited to be honored by the union.

I find it difficult to write, so it’s very encouraging and exciting to get an award,” Coppola told Reuters after the event.

Coppola’s competition was Gurinder Chadha, Paul Mayeda Berges and Guljit Bindra for Bend It Like Beckham, Steven Knight for Dirty Pretty Things, Irish director Jim Sheridan and his daughters Naomi and Kirsten for In America, and first-time writer / director Tom McCarthy for The Station Agent.

The other adapted screenplay nominees were director Peter Jackson, Fran Walsh and Philippa Boyens for The Lord of the Rings: The Return of the King, Anthony Minghella for Cold Mountain, Brian Helgeland for Mystic River, and writer / director Gary Ross for Seabiscuit.

Coppola, Knight and the Sheridans will vie for the Academy Award next week, along with the writers of The Barbarian Invasions and Finding Nemo.

Apart from Cold Mountain, all the Writers Guild of America adapted screenplay contenders will compete for the Oscar, along with the Brazilian drama City of God.

In the last 12 years, eight of the Writers Guild of America adapted screenplay winners and seven of its original screenplay winners have gone on to Oscar glory.

Lost in Translation has already picked up three Golden Globes—an Oscar bell-wether—including best screenplay. The Lord of the Rings: The Return of the King, which has 11 Oscar nominations, has collected four Golden Globes and prizes from Hollywood’s producers and directors guilds.

Questions:

67. What does American Splendor mainly write about?

68. Sofia Coppola’s Lost in Translation won the Writers Guild of America Award for________.

69. The Writers Guild of America Awards were offered at the same time in________and________.

70. How did Coppola feel about her getting the award?

71. List at least 3 movies that competed with Lost in Translation.

72. How many Writers Guild of America screenplay winners have gone on to Oscar glory in the last 12 years?

Questions 73 to 78 are based on the following passage:

The need for a surgical operation, especially an emergency operation, almost always comes as a severe shock to the patient and his family. Despite modern advances, most people still have an irrational fear of hospitals and anaesthetics.

In the early years of last century there was little specialization in surgery. A good surgeon was capable of performing almost every operation that had been devised up to that time. Today the situation is different. Operations are now being carried out that were not even dreamed of fifty years ago. The heart can be safely opened and its valves repaired. Clogged blood vessels can be cleaned out, and broken ones mended or replaced. A lung, the whole stomach, or even part of the brain can be removed and still permit the patient to live a comfortable and satisfactory life.

The scope of surgery has increased remarkably in 20th century. Its safety has increased too. Deaths from most operations are about 20% of what they were in 1910 and surgery has been extended in many directions,for example to certain types of birth defects in newborn babies, and, at the other end of the scale, to life-saving operations for the octogenarian.The hospital stay after surgery has been shortened to as little as a week for most major operations.

Many developments in modern surgery are almost incredible. They include the replacement of damaged blood vessels with simulated ones made of plastic; the replacement of heart valves with plastic substitutes; the transplanting of tissues such as the lens of the eye.

One of the most revolutionary areas of modern surgery is that of organ transplants. Until a few years ago, no person, except an indentical twin, was able to accept into his body the tissues of another person without reacting against them and eventually causing death. Recently, however, it has been discovered that with the use of x-rays and special drugs, it is possible to graft tissues from one person to another which will survive for periods of a year or more. Kidneys have been successfully transplanted between non-identical twins. Heart and lung transplants have been reasonably successful in animals, though rejection problems in humans have yet to be solved.

Spare parts” surgery, the simple routine replacement of all worn-out organs by new ones, is still a dream of the distant future. As yet, surgery is not ready for such miracles. In the meantime, you can be happy if your doctor says to you,“Yes, I think it is possible to operate on you for this condition.”

Questions:

73. Most people are afraid of being operated on in spite of________.

74.A patient can still live a comfortable and satisfactory life even after the removal of________.

75. Today deaths from most operations are about ________of what they were in 1910.

76. What’s the main difficulty in organ transplanting?

77. Is “spare parts” surgery possible now?

78. You can be happy if your surgeon can operate because it means________.

Questions 79 to 84 are based on the following passage:

Sales of anti-ageing skin treatments have reached a new high as British women try to stay younger longer (writes Joanna Bale). But according to a recent survey of those aged between 35 and 55 there are significant regional variations in annual spend on these cosmetics.

While the average woman thinks costs of 200 a year acceptable—almost treble the 75 of three years ago—some fork out 500, according to the survey of over 2,000 women nationwide by the beauty company Olay.

Brows will wrinkle at the differences in yearly spend on anti-ageing treatments across major cities: the ladies of Edinburgh spent least, just 50 a year, while those in Leeds spend most, at a staggering 500. London women most commonly spend 200 annually, and those in Manchester give themselves a modest yearly budget of 100.

Equally surprising are the results among 40-somethings. Single women spend the least on indulging their desire for facial rejuvenation, with only 25 per cent forking out on skincare products or treatments. This figure rises to just over 31 per cent of married women and a similar figure for those who are unmarried with partners.

By far the biggest spenders are the 50 per cent of divorcees who feel the pressure to invest heavily in their facial futures.

The survey highlighted two groups who spend significant sums on enhancing their appearances—“Sindies” (single income now divorced), and women in their 40s who use their looks to get ahead.

The survey also found that although women wanted to “de-age” they had a holistic view of beauty and embraced a realistic and natural approach to looking good.

Questions:

79. Compared with that of three years ago, the average British woman’s annual spend on cosmetics has almost________.

80. Women in________spend most on cosmetics according to the survey.

81. The ladies of Birmingham as well as those in ________spend 100 a year on antiageing treatments.

82. ________and________spend more on indulging their desire for facial rejuvenation than single women.

83. Enhancing appearances plays an important role in the life of________and________.

84. After reading the passage, what do you learn about the sales of anti-ageing skin treatments now in Britain?

Questions 85 to 90 are based on the following passage:

It is hard to get any agreement on the precise meaning of the term “social class”. In everyday life, people tend to have a different approach to those they consider higher or lower than themselves in the social scale. The criteria we use to “place” a new acquaintance, however, are a complex mixture of factors. Dress, way of speaking, area of residence in a given city or province, education and manners all play a part.

In ancient civilizations, the Sumerian, for example,social differences were based on birth,status or rank,rather than on wealth. Four main classes were recognized. These were the rulers, the priestly administrators, the freemen (such as craftsmen,merchants or farmers) and the slaves.

In Greece, after the sixth-century B.C., there was a growing conflict between the peasants and the landed aristocrats, and a gradual decrease in the power of the aristocracy when a kind of “middle class” of traders and skilled workers grew up. The population of Athens,for example, was divided into three main classes which were politically and legally distinct. About one-third of the total were slaves, who did not count politically at all, a fact often forgotten by those who praise Athens as the nursery of democracy. The next main group consisted of resident foreigners, the “metics”, who were freemen, though they too were allowed no share in political life. The third group was the powerful body of “citizens”, who were themselves divided into subclasses.

In ancient Rome, too, a similar struggle between the plebs, or working people, and the landed families was a recurrent feature of social life.

The medieval feudal system, which flourished in Europe from the ninth to the thirteenth centuries, gave rise to a comparatively simple system based on birth.Under the king there were two main classes—lords and “vassals”, the latter with many subdivisions. The vassal owed the lord fidelity, obedience and aid, especially in the form of military service. The lord in return owed his vassal protection and an assured livelihood.

In the later Middle Ages, however, the development of a money economy and the growth of cities and trade led to the rise of another class, the “burghers” or city merchants and mayors. These were the predecessors of the modern middle classes. Gradually high office and occupation assumed importance in determining social position, as it became more and more possible for a person born to one station in life to move to another. This change affected the towns more than the country areas, where remnants of feudalism lasted much longer.

Questions:

85. List at least three common criteria for telling a person’s social position.

86. What were the four main classes in the Sumerian civilization?

87. Slaves in Greece in the sixth century B.C. were not________significant.

88. The struggle between the plebs and the landed families was a________feature of social life.

89. The metics,one of the three classes of Greece, consisted mainly of________.

90. What did the development of a money economyand the growth of cities and trade lead to?

Part V Error Correction (5 minutes,10 points)

Directions:The following passage contains 9 errors. In each case only one word is involved. You should proofread the passage on the Answer Sheet and correct it in the following way:

EXAMPLE

One night,quite late,I was still awake in the room I am shared with 1. am

my husband. I was lying on my right side and can hear a child crying. 2. could

Getting up,I went see if our son was all right. 3. to

He was sleeping soundly,breathing deeply and gently. 4.

If the air in New York seems a little less grimy this spring, thank Rudolph Giuliani. On January 10th, after months of burning debate, the city’s non-smoke mayor

91.________

signed the Smoke-Free Air Act. From April 10th smoking will be stubbed out(碾灭) in restaurants catering for more than 35 people, a move that will hit about half the citys 11,000 eating places. Nicotine addicts will also smoked out at work, except

92.________

in ventilated smoking rooms or offices occupied by no more than three consenting adults. More radically, outdoor seating areas will also become smoke zones.

93.________

Come the new baseball season, fans at Yankee Stadium will be breaking the law if they light up.

New York joins well over 100 American cities—and four states—that have passed laws banned smoking

94.________

in public places. More than a third of American companies now forbid smoking in the workplace, up to

95.________

a mere 20% in 1986. And the tobacco industry, which in America alone has annual sales of close to 50 billion, is watching its profits go down in smoke.

96.________

The industry may never recover. Polls suggest that nine out of ten Americans are irritated by cigarette smoke. With good reason. In 1993 the Environmental Protection Agency (EPA) has classified

97.________

second-hand” smoke as a health hazard—one that,according to the EPA, causes 3,000 non-smokers to die from lung cancer each year.

98.________

New Yorkers must now wait and see if the pro-smoking lobby’s alarming predictions of citywide economical collapse come true. Tobacco

99.________

company Philip Morris may show the way. Last year it threatened to move its 2,000 head-office employee out

100.________

of the city if the smoking ban became law.

Part VI Translation (10 minutes, 20 points)

Section A English-Chinese Translation (10 points)

Directions: Translate the underlined sentences of the following passage into Chinese on the Answer Sheet.

What is a brand anyway? A brand isn’t just a logo on a shirt or an ice-cream van. It’s much more than that. To be successful a brand has to have rational characteristics—(101)ie, it has to be competitive on price or quality or service, and it has to have a “personality” that charms and seduces. It is a mix of emotional factors,such as “Do I like it?” and “Is it me?” and rational factors such as,“Is it cheaper or better or quicker?”(102)Getting things in balance is tricky,and that’s why so many brands don’t succeed.

Just let’s look at all the ways brands can fail. Brands are vulnerable to fashion. Fizzy drinks such as Coke and Pepsi are now being attacked by stimulation drinks such as Red Bull. Fast food brands are threatened by salads and other “lite” foods. McDonalds has recently been in real trouble.

Brands are also vulnerable because they get cocky,arrogant and out of touch. They think they know best and don’t change with the market—like the Gap, Levi’s and Marks & Spencer brands who learnt their lesson the hard way.

(103)In fact, even the people who create brands can’t really control them or even predict how people will use them. Not one single mobile phone company anywhere in the world anticipated the growth of texting.The companies involved in the new 3G phone technology are still holding their breath because they don’t know whether it will take off or not. And if it does take off, they won’t know how and in what direction until the market tells them. It’s a huge gamble.Range Rover was the first Sport Utility Vehicle, but Land Rover never completely understood the concept that it had inspired. Range Rover has spent much of its life span trying to catch up with the trend that it serendipitously created. (104)Organic foods are a great success, but not one single major manufacturer or retailer originally promoted them. Little companies started the trend and we consumers just decided we didn’t want our food mucked around with. So at first slowly, hesitantly and ponderously, the retailers and the manufacturers followed. They did what we asked.

(105)In other words, brands are nothing like as powerful as they look. The people who manage them often get things wrong, muck things up, look in the wrong direction and generally act just like most organisations directed by human beings—messily. So despite all the huffing and puffing, the reality is that brands are more or less completely in our power. When we like them we buy them, when we don’t, we just buy something else. And what’s more, as customers we’re unpredictable. We can be loyal or fickle, extravagant or stingy, serially or simultaneously.

Section B Chinese-English Translation (10 points)

106. 许多遭到洪水侵害的农场主说,他们别无选择只得解雇一些工人。

107. 有时候对一个人来说需要终生的时间才能懂得活着就是为了奉献。

108. 在美国,由于道路畅通,开车上下班很方便,现在许多在城市里工作的人,喜欢住在乡下。

109. 收音机及电视机使得做广告的人有可能用这种方法吸引千百万人的注意。

110. 纽约劳动力市场近来显现出从二战以来最长时间的低迷中复苏的迹象。

Part VII Writing (30 minutes, 30 points)

Writing Task I (10 points)

Directions: Yesterday you lost your student identification card. You need it to get discounts on public transport and for the cinema, and to use the college library. Even more importantly, you need it as proof of identity to withdraw money from the bank. Write to the Director of Student Services explaining the situation and requesting a new card as soon as possible.

You should write about 120 words. You do NOT need to write your own address. Write your letter on the Answer Sheet. Begin your letter as follows:

Dear ________,

Writing Task II (20 points)

Directions: The following is an advertisement for English teachers.Suppose you are a university graduate,and your future plan is to be an English teacher. After reading this advertisement, you decide to write a statement of interest to GEOS. You should write at least 150 words. Write your statement on the Answer Sheet.

2004年全国大学生英语竞赛初赛 听力录音原文及参考答案

Part I Listening Comprehension (30 minutes, 30 points)

Section A Dialogues (10 points)

Directions: In this section, you will hear 10 short dialogues. At the end of each dialogue, a question will be asked about what was said. Both the dialogue and the question will be read only once. After each question,there will be a pause. During the pause, you must read the four choices marked A, B, C and D, and decide which is the best answer. Then mark the corresponding letter on the Answer Sheet with a single line through the centre.

1. W: Hi, I’d like to send this package by express mail to San Francisco and I would like to buy a sheet of stamps, please.

M: Here are your stamps, and just put the package on the scale.

Q: Where did the conversation take place? (D)

2. M: I’m going out to lunch. Do you need anything while I’m out?

W: Yes, if you pass a convenience store, get me some chocolate—a Snickers bar, please.

Q: What do you learn from this conversation? (B)

3. W: If we go by car, how do we cross the river?

M: There’s a ferry that will take your car. There’s even one for trains.

Q: How will they cross the river? (D)

4. W: I heard that the mayor is closing the cheese factory.

M: Yes, but it is only temporary.

W: Oh, I’m surprised. I thought it was going to shut down for good.

Q: Why was the woman surprised? (C)

5. M: I spilled tomato juice on my new white shirt. Do you think it will come out?

W: That’s too bad. Leave it there and I’ll see what I can do.

Q: What is the man’s problem?(B)

6. W: I’m going to lunch with my bowling instructor.

M: What about the committee meeting?

W: Don’t worry. I’ll be back at the office before then.

Q: Where is the woman probably going now? (C)

7. M: How long have you had this problem with your shoulder?

W: It started last week after my skiing accident.

M: Let’s try some tests to determine the nature of the injury.

Q: What is the man going to do? (B)

8. W: Are you having a good time?

M: Sure. Thanks again for inviting me.

W: No problem. I just wish more people could have come.

Q: How does the woman feel? (C)

9. M: We finally made it, Mary!

W: I can’t believe graduation is tonight.

M: Can you come to my graduation party?

W: Sure, after I finish the family celebration.

M: I want to be sure we get pictures of us together.

W: In our caps and gowns!

Q: When will the woman go to the man’s graduation party? (A)

10. M: Hi, did you pass your geography exam?

W: Yeah, I did quite well in fact, I got 76%.

M: Oh,well done! So they gave you a per cent? I thought they gave grades.

W: Yeah, they gave both. Mine was an “A”. So how about you?

M: Well, we don’t have exams.We have continuous assessment, so you just have to do coursework, and you get a mark for each essay.

Q: How does the school evaluate the man’s progress in geography? (A)

Section B News Items (10 points)

Directions: In this section, you will hear 10 short pieces of news from BBC or VOA. After each news item and question,there will be a pause. During the pause, you must read the three choices marked A, B and C, and decide which is the best answer. Then mark the corresponding letter on the Answer Sheet with a single line through the centre.

11. Tens of thousands of health workers will go house to house over the next three days in an effort to immunise 63 million children under the age of five in sub-Saharan Africa. The campaign is the start of monthly national immunisation days during the low season for polio. It’s hoped that vaccinating children now—when the virus is at its weakest—will be the best way of stopping transmission.

Question:How old are the children to be immunised?(B)

12. Amid pomp and ceremony, China launched the 2008 Olympics. Together with a Chinese counterpart, the president of the International Olympic Committee, Jacques Rogge, used a giant golden key to symbolically open what he called the most important market in the world. In his speech, he emphasised the power of the Olympic brand in China’s emerging market.

Question:What does the giant golden key symbolize?(C)

13. Microsoft tries to keep the code for its Windows operating system a closely guarded secret. It’s the equivalent of computer DNA and the firm fears if it falls into the wrong hands it could be used to infiltrate millions of computers worldwide. More than 90 percent of the world’s PCs run Windows.

Question:What action does Microsoft intend to take?(A)

14. Before he set off in November, there were fears that Francis Joyon would be unable to control his huge boat, named IDEC. With its three hulls slicing through the water and a massive rotating mast that reached 30 metres into the sky, the boat was built in 1986 for a crew of ten. It was feared that such a boat would be too powerful for one man in the rough seas of the Southern Ocean.

Question:How many people can the boat carry?(B)

15. Over timescales of thousands of years, the Earth goes through a natural cycle of warmer and colder periods, driven by changes in heat coming from the Sun. Professor William Ruddiman from the University of Virginia has now calculated that if the Earth had followed its natural cycle over the last ten thousand years, it ought to have got steadily colder. It hasn’t,because, he believes, human activities have been keeping the temperature steady.

Question:Has the Earth got steadily colder over the last ten thousand years?(A)

16. Inequality of health care is still paramount, says the WHO’s latest report. Industrialised countries account for less than 20 percent of the world’s population but take 90 percent of health spending. In Japan more than 500 dollars is spent on drugs per person per year. This compares to just three dollars in Sierra Leone. Only slightly more is spent in many sub-Saharan countries.

Question: How much do many sub-Saharan countries spend on drugs per person per year?(B)

17. The Iraqi dinar has risen a third or so in value against the dollar since the new banknotes began to circulate. One factor has been the gradual pick up of the Iraqi economy after the devastation of the war. There are simply more transactions taking place, which has supported the value of the currency. And it seems Iraqis trust the new dinar banknotes more than they did the old ones, which featured pictures of Saddam Hussein.

Question:Why did the Iraqi new dinar rise in value?(C)

18. The list of countries known to have the relatively new and deadly strain of bird flu is rapidly growing. The focus now is on Indonesia where tests will soon confirm whether or not the bird flu which killed several million chickens there is the often fatal H5N1, already confirmed in 5 other countries in the region. Reports of an outbreak in Laos are also being investigated.

Question:What is the number of countries mentioned in this news report?(C)

19. An unhealthy diet together with little exercise and smoking are the key preventable risks of non-communicable diseases and it’s estimated that low fruit and vegetable intake alone causes more than two and a half million deaths each year.

Question:What causes more than two and a half million deaths each year?(A)

20. Around Europe interest rates are at their lowest levels in half a century. But businesses are pressing for even cheaper borrowing costs amid signs of continued economic weakness.

A big drop in German manufacturing announced earlier this week is cited as evidence that Europe’s most important economy may even be sliding into recession. And the rise of the euro to a four-year high against the dollar in currency dealing is a major worry for many European exporters.

Question:What is the key problem for European exporters?(A)

Section C Passages (10 points)

Directions:In this section, you will hear 2 passages. At the end of each passage, you will hear 5 questions. After you hear a question, you must choose the best answer from the four choices marked A, B, C and D. Then mark the corresponding letter on the Answer Sheet with a single line through the centre.

Passage One

The world of music will never be the same since the formation of a band in Liverpool, England in 1956. The Beatles were formed by George Harrison, Ringo Starr, Paul Mc-Cartney, and John Lennon. Their first hit song Love Me Do was recorded in 1962. The Beatles quickly became the world’s best-known pop music group and many people today still regard them as the finest band in the history of pop music.

Lennon and McCartney were the authors of most of the songs the group recorded. Harrison also wrote songs, often using ideas from Indian music. The drummer of the group was the famous Ringo Starr and he occasionally sang. For six years the Beatles had hit after hit song. Twenty-eight of their songs were on the Top Twenty record charts and seventeen of these songs reached number one on the charts.

The group also had a successful movie career. The comedies A Hard Day’s Night and Yellow Submarine became very successful movies. People imitated their long hairstyles, clothing, and humor. Almost all later pop bands learned from the Beatles. Beatlemania is the word used to describe how strong and loyal the fans were.

Questions 21 to 25 are based on the passage you have just heard:

21. What kind of music did the Beatles play?(D)

22. What did many people copy from the Beatles?(D)

23. Where were the members of the Beatles group from?(B)

24. Which of the following is NOT true?(C)

25. How many of the Beatles’ songs reached number one on the record charts?(A)

Passage Two

Have you ever wondered where these cute little teddy bears came from? They were named for President Theodore Roosevelt in 1902.

President Roosevelt was on a hunting trip in Mississippi when members of the hunting party caught a black bear and tied him to a tree. President Roosevelt was called to the area to shoot the bear, which he refused to do and said it was unsportsmanlike and showed poor manners.

The Washington Post newspaper ran a cartoon showing the President refusing to shoot the bear and people all over America saw the cartoon.

Morris Michtom, a shopkeeper in Brooklyn, New York, placed two toy bears in the window of his shop. Mr. Michtom requested permission from the President to call them “Teddy Bears” as Teddy is the nickname for Theodore Roosevelt. The sweet little bears with shiny button eyes were a delight with children everywhere. The Teddy Bears were made by Mr. Michtom’s wife. Mr. Michtom formed a new business called the Ideal Novelty and Toy Corporation.

Today, Teddy Bears are treasured toys of children all over the world. They are also collected by people and many are displayed in museums. Teddy Bears are sold by many companies and you can find them in almost any toy store, dressed in costumes or with a ribbon around the neck.

Questions 26 to 30 are based on the passage you have just heard:

26. Why did President Roosevelt refuse to shoot the bear?(C)

27. Why did Mr. Michtom ask for the President’s permission to call the toy bears “Teddy Bears”?(A)

28. Which of the following is NOT true?(D)

29. How many Teddy Bears were made by Mrs. Mitchtom and placed in the window of their shop?(C)

30. What did Mr. Mitchtom do after he sold the Teddy Bears in 1902?(D)

Part II Vocabulary and Structure (10 minutes, 20 points)

Section A Multiple Choice (10 points)

31. A 32. D 33. A 34. C 35. B 36. C 37. B 38. D 39. B 40. C

Section B Cloze-Test (10 points)

41. B 42. A 43. A 44. B 45. A 46. B 47. C 48. D 49. D 50. A

Part III Word Guessing and IQ Test (5 minutes, 10 points)

Section A Word Guessing (5 points)

51. B 52. B 53. D 54. C 55. B

Section B IQ Test (5 points)

56. C 57. A 58. B 59. A 60. A

Part IV Reading Comprehension (25 minutes,30 points)

61. trays

62. To preserve their colours. (or: To prevent darkening.)

63. In hot-air chambers.

64. dried separately and then mixed

65. climbers, explorers, soldiers

66. Because it takes so little time to cook them.

67. The travails of comics connoisseur Harvey Pekar.

68. original screenplay

69. Los Angeles, New York

70. Encouraged and excited.

71. Bend It Like Beckham, Dirty Pretty Things, In America, The Station Agent. ( Any three of them.)

72. 15.

73. modern advances in surgery

74. the stomach or one lung

75. 20%

76. The body’s tendency to reject alien tissues.

77. No, it has yet to become a reality.

78. your illness may be curable

79. tripled

80. Leeds

81. Manchester

82. Married women, those unmarried with partners

83. “Sindies”, women in their 40s

84. The sales have reached a new high, with regional variations.

85. Dress, way of speaking, area of residence education and manners. (Any three of them.)

86. Rulers, administrators, freemen and slaves.

87. politically

88. recurrent

89. resident foreigners

90. The rise of the burghers.

Part V Error Correction (5 minutes,10 points)

91. non-smokenon-smoking

92. also smokedbe

93. smokesmokeless / non-smoking

94. bannedbanning

95. tofrom

96. downup

97. hashas

98.

99. economicaleconomic

100. employeeemployees

Part VI Translation (10 minutes, 20 points)

Section A English-Chinese Translation(10 points)

101. 即它必须在价格或质量或服务方面具有竞争力,并且还应具有能够吸引人们购买的个性特点

102. 保持诸方面因素的平衡是很需要技巧的,这也正是如此多的品牌不成功的原因所在。

103. 事实上,即使是设计品牌的人也无法左右品牌,他们甚至预测不出人们将如何利用它们。

104. 有机食品是很成功的, 但在最初却没有一个较大规模的生产商或零售商(愿意)推销它们。

105. 换句话说,品牌本身远没有他们看起来那样有影响力。

Section B Chinese-English Translation (10 points)

106. Many flood-ravaged farmers say they will have no choice but to lay off some employees.

107. Sometimes it takes a lifetime for one to realize that to live is to give.

108. Many people in America who work in the cities now enjoy living in the country because of good roads and the ease of commuting by automobile.

109. Radio and television have made it possible for advertisers to capture the attention of millions of people in this way.

110.The labor market of New York has recently shown signs of emerging from its longest slump since World War II.

Part VII Writing (30 minutes, 30 points)

Writing Task I (10 points)

One possible version:

Dear Director,

I have to report the loss of my student identification card yesterday, Tuesday, September 3rd. I am a student in Business Studies. I believe I lost my card at the swimming pool when I dropped my backpack and many items fell to the ground.

I need my card to get money from the bank. The bank will only accept my student card as ID. I asked the bank if they would accept my passport, but they said that my student ID card was used to open the account, and that I should bring it with me when I want to withdraw money.

I also want to go to the cinema, but I will not get a concession without my student card.

Could you please issue me a new card as soon as possible? I will come to your office this afternoon to see if I can get one immediately.

Thank you very much.

Yours sincerely,

###

Writing Task II (20 points)

One possible version:

Dear Sir or Madam:

I am currently a student at # # University. After reading your message in # # newspaper, I am very interested to apply for a teaching position with GEOS.

English has become a world language, spoken and understood everywhere in the world. Therefore, to be an English teacher in foreign countries is to experience different cultures. Some of my friends, who are from the UK and are teaching English here, regard teaching English not only as a career but also a chance to learn about people and culture. They set good examples for me, and they have encouraged me to apply for a chance to join you.

English is one of my favorite subjects. To be an English teacher always has been my dream. After four years of study in this university, I have passed the College English Test Band 6. Most of the courses I have taken are specially focused on teaching methods and skill training.

If you are looking for an energetic, outgoing and motivated university graduate, I am sure you have found one.

Sincerely yours,

###

2008年全国大学生英语竞赛样题(C)

 Part IListening Comprehension (25 minutes, 30 marks)

Section A(5 marks)

  Directions: In this section, you will hear 5 short conversations. At the end of each conversation, a question will be asked about what was said. Both the conversation and the question will be read only once. After each question, there will be a pause. During the pause, you must read the three choices marked A, B and C, and decide which is the best answer. Then mark the corresponding letter on the Answer Sheet with a single line through the centre.

  1. A. The man is not suitable for the position.

  B. The job has been given to someone else.

  C. She hadn't received the man's application.

  2. A. He is going to see his section chief.

  B. He is going to have a job interview.

  C. He is going to see his girlfriend.

  3. A. Ask to see the man's ID card.

  B. Get the briefcase for the man.

  C. Show the man her documents.

  4. A. The dorm room is too crowded.

  B. There is no kitchen in the building.

  C. No one looks after the dorm building.

  5. A. She was always in good shape.

  B. She stopped exercising one year ago.

  C. She lost a lot of weight in one year.

  Section B (10 marks)

  Directions: In this section, you will hear two long conversations. Each conversation will be read only once. At the end of each conversation, there will be a one-minute pause. During the pause, you must read the five questions, each with three choices marked A, B and C, and decide which is the best answer. Then mark the corresponding letter on the Answer Sheet with a single line through the centre.

  Conversation One

  6. What is soon to open in China?

  A. The French Movie Festival.

  B. The French Cultural Year.

  C. The French Food Festival.

  7. How many exhibitions will be held for this activity?

  A. 200.B. 20.C. 100.

  8. What will be held at the foot of the Great Wall?

  A. The City Concert.

  B. The Opening Ceremony.

  C. The Great Lunch.

  9. Which of the following cities is not included in this activity?

  A. Chongqing.

  B. Wuhan.

  C. Shenzhen.

  10. What will certainly make great contributions to this activity?

  A. Internet.

  B. TV shows.

  C. Newspapers.

  Conversation Two

  11. What sound more like a native speaker in a casual conversation?

  A. Examples.  B. Verbs.  C. Idioms.

  12. Which of the following sounds more informal and more natural?

  A. Get together.  B. Meet.  C. See.

  13. Which of the following means that you cannot interrupt me?

  A. I'm tied up.

  B. I have a lot on my plate.

  C. I'm busy.

  14. In American culture, what is considered important in a conversation?

  A. Using proper language

  B. Making eye contact.

  C. Looking at your own feet.

  15. In business, how might Americans feel about you if you are looking away?

  A. You're feeling ashamed.

  B. You're telling the truth.

  C. You're telling a lie.

  Section C (5 marks)

  Directions: In this section, you will hear 5 short news items. After each item, there will be a pause. During the pause, you must read the question and then the three choices marked A, B and C, and decide which is the best answer. Then mark the corresponding letter on the Answer Sheet with a single line through the centre.

  16. How long will it take for new forests to grow back?

  A. 65 years.  B. 20 years.  C. 40 years.

  17. What did Jimmy Carter plan to focus his efforts on after leaving the White House in 1981?

  A. A presidential library.

  B. Camp David.

  C. Winning a second term.

  18. What was regarded as the lifeblood of the country of the Maldives?

  A. Oil.  B. Agriculture.  C. Tourism.

  19. What is responsible for the death of many people in developing countries?

  A. The development of resistance to diseases.

  B. The difficulty to cure new emerging diseases.

  C. The inability of the poor to afford medicine.

  20. What released an estimated 8.7 million tons of the global warming gas?

  A. Cars.  B. Wildfires. C. Wars.

  Section D (10 marks)

  Directions: In this section, you will hear a short passage. There are ten missing words or phrases in it. Fill in the blanks with the exact words you hear on the tape. Remember to write the answers on the Answer Sheet.

  A researcher says lead in the environment could be a major cause of violence by young people. Doctor Herbert Needleman is a (21) ________ at the University of Pittsburgh School of Medicine in Pennsylvania and he (22) ________ his findings at the yearly meeting of the American Association for the Advancement of Science. Doctor Needleman says the presence of lead in the (23) ________ changes the neurons that control actions and that can cause a person to act in antisocial and (24) ________ ways.

  In the 1970s, Doctor Needleman found lower scores on (25) ________ even in children who did not have such signs of lead poisoning. After that, lead was (26)________ gasoline and paint in the United States. Yet many homes still have old lead paint. Lead was also used in older (27) ________. In fact, officials just announced stronger testing and reporting requirements as from next year for lead in American drinking water.

  The newest research shows that even very small amounts of lead in bones can affect brain development. A simple (28)________ can measure lead except that an X-ray process is needed to measure levels in bone. In 2004, such tests were done on 190 young people who were (29) ________ and the findings showed that their average levels were higher than normal. And, in 1998, three hundred children were studied and the test scores showed higher levels of (30) ________ problems in those with increased levels of lead. Yet these levels were still considered safe by the government.

Part II Vocabulary and Structure (10 minutes, 15 marks)

Directions: There are 15 incomplete sentences in this part. For each blank there are four choices marked A, B, C and D. Choose the one that best completes the sentence. Then mark the corresponding letter on the Answer Sheet with a single line through the centre.

  31. So nervous ________ that she didn't know how to start her speech.

  A. since she became

  B. would she become

  C. that she became

  D. did she become

  32. He ________ another career but, at the time, he just wanted to earn money to study abroad.

  A. might have chosen  B. might choose

  C. had to choose  D. must have chosen

  33. The second report was ________ by August 2005, but one year later it was still nowhere in sight.

  A. submitted  B. to have submitted

  C. to submit  D. to have been submitted

  34. In this experiment, the students studied are stopped several times during the listening test and asked to report what they ________ during the pause before answering the questions.

  A. had just been thinking about  B. have just been thinking about

  C. are just thinking about  D. had just thought about

  35. I was always taught that it was ________ to interrupt.

  A. rude  B. coarse  C. rough  D. crude

  36. Small boys are ________ questioners. They ask questions all the time.

  A. original  B. peculiar  C. imaginative  D. persistent

  37. We regret to inform you that the materials you ordered are ________.

  A. out of work  B. out of reach  C. out of stock  D. out of practice

  38. The bomb will ________ the moment it is touched.

  A. go on  B. go off  C. go out  D. go over

  39. The car won't ________; I've tried it several times, but it won't work.

  A. begin  B. launch  C. start  D. drive

  40. Children and old people do not like having their daily ________ upset.

  A. habit  B. routine  C. practice  D. custom

  41. In your first few days at school you'll be given a test to help the teachers to ________ you to a class at your level.

  A. locate  B. assign  C. deliver  D. place

  42. China only started its nuclear power industry in recent years, and should ________ no time in catching up.

  A.lose B.delay C. spare D. relieve

  43. — You did an excellent job yesterday, Jim! I really enjoyed your presentation.

  — ________

  — Oh yeah, it was fabulous. It seems the English program is a great way to practice English.

  — Yeah. It is fun and motivating.

  A. Did you really? 

  B. Oh, thank you. You are so kind.  

  C. Really? What about yours? 

  D. Not at all. My pleasure.

  44. — What kind of music do you like?

  — Well, I like different kinds.

  — ________

  — Er, I especially like punk rock.

  A. I beg your pardon? B. Are you serious?

  C. Any in particular? D. Why do you think so?

  45. — How did you like the fashion show last night?

  — ________

  — I didn't see anything wrong with the clothes; they looked pretty nice to me.

  — Do you really think people can wear that stuff and walk around in streets?

  A. Impressive. It's a good way to show off women's sense of style and wealth.

  B. It was cool. The clothes are more beautiful than the people wearing them.

  C. Nothing serious. It's only a show to attract the eyes of fashion fans.

  D. It was dumb. I think it's stupid for women to wear clothes like that.

Part III Reading Comprehension (20 minutes, 40 marks)

Section A (4 marks)

  Directions: There is one passage in this section with 4 questions. For each question, there are four choices marked A, B, C and D. You should decide on the best choice. Then mark the corresponding letter on the Answer Sheet with a single line through the centre.

  Questions 46-49 are based on the following passage.

  The ability to see oneself in the future is a remarkable human trait - some would say unique - that is not well understood. That's despite the fact that we probably spend as much time thinking about the future as we do thinking about the present.

  Now new research from Washington University in St. Louis suggests that it's precisely because we can remember the past that we can visualize the future. Our findings provide convincing support for the idea that memory and future thought are highly interrelated and help explain why future thought may be impossible without memories,” says doctoral candidate Karl Szpunar. The findings are consistent with other research showing that persons with little memory of the past, such as young children or individuals suffering from loss of memory, are less able to see themselves in the future.

  The researchers base their conclusions on brain scans of 21 college students who were cued to think about something in their past, and anticipate the same event in the future, like a birthday or getting lost. The experiment was carried out as each student lay on their stomach in a magnetic resonance imaging machine, a dreadful but very useful piece of equipment that can show which areas of the brain are stimulated during specific thought processes.

  The students were also asked to picture former President Bill Clinton in a past and future setting. Clinton was chosen because he was easily recognized and familiar to all the students.

  The researchers found a surprisingly complete overlap among regions of the brain used for remembering the student's past and those used for picturing the future. And every region involved in remembering was also used in anticipating the future.

  In short, the researchers isolated the area of the brain that lit up when the students thought about an event in their own past. And more importantly, that same area lit up again when they thought about a similar event in their future. In fact, the researchers report that the brain activity was so similar in both cases that it was “indistinguishable.”

  The findings were reinforced when students imagined Bill Clinton. Since none of them knew him personally, their memories were not autobiographical. And the brain scans showed significantly less correlation between memories of having seen pictures of Clinton in the White House and projecting him into the future.

  So this time machine, as the researchers describe it, allows us to use the past to see ourselves in the future, and both our memories and our anticipation are interdependent.

  46. A remarkable human trait that is not well understood is the ability ________.

  A. to think about the past B. to see the future C. to remember the past D. to control the present

  47. The findings support that ________.

  A. future goals will greatly influence a person's present performance

  B. a person's present performance is determined by his / her past knowledge

  C. future thought depends to a great degree on the memory of the past

  D. present thought is impossible without the ability to imagine the future

  48. The conclusion of the experiment on students was that ________.

  A. the students could picture themselves better than Bill Clinton in a past and future setting

  B. the students could imagine themselves as well as Bill Clinton in a past and future setting

  C. the students could anticipate Bill Clinton better than themselves in a past and future setting

  D. the students could only picture themselves in a past and future setting but not Bill Clinton

  49. This time machine in the last paragraph most probably refers to ________.

  A. clock B. brain scanning C. magnetic resonance imaging D. memory

  Section B (14 marks)

  Directions: There is one passage in this section with 10 questions. Go over the passage quickly and answer the questions on the Answer Sheet. For questions 50-55, mark Y (for YES)if the statement agrees with the information given in the passage; N (for NO)if the statement contradicts the information given in the passage; NG (for NOT GIVEN)if the information is not given in the passage. For questions 56-59, complete the sentences with the information given in the passage.

  Visiting the White House

  White House Tours Public tours of the White House are available for groups of 10 or more people. Requests must be submitted through one's Member of Congress and are accepted up to six months in advance. These self-guided tours are available from 7:30 a.m. to 12:30 p.m. Tuesday through Saturday , and are scheduled on a first come, first served basis approximately one month in advance of the requested date. We encourage you to submit your request as early as possible since a limited number of tours are available. All White House tours are free of charge. For the most current tour information, please call the 24-hour line at 202-456-7041. Please note that White House tours may be subject to last minute cancellation.

  White House Visitor Center

  All tours are significantly enhanced if visitors stop by the White House Visitor Center located at the southeast corner of 15th and E Streets, before or after their tour. The Center is open seven days a week from 7:30 a.m. until 4:00 p.m. and features many aspects of the White House, including its architecture, furnishings, first families, social events, and relations with the press and world leaders, as well as a thirty-minute video. Allow between 20 minutes to one hour to explore the exhibits. The White House Historical Association also sponsors a sales area. Please note that restrooms are available, but food service is not.

  Mobility-Impaired / Using a Wheelchair

  Guests requiring the loan of a wheelchair should notify the officer at the Visitors Entrance Building upon arrival.

  Wheelchairs loans are offered on a first-come, first-served basis. Reservations are not possible.

  Visitors in wheelchairs, or with other mobility disabilities, on the Congressional guided or self-guided tours, between 8:00 a.m. and 12 noon, use the same Visitor entrance and, with up to four members of their party, are admitted without waiting in line and without tickets.

  Visitors in wheelchairs are escorted by ramp from the entrance level to the ground floor, and by elevator from the ground to the state floor. Guests generally wait in line with their family or group.

  Hearing-Impaired

  Tours for hearing-impaired groups may be arranged in advance by writing to the Visitors Office, White House, Washington, DC 20502. Tours are usually scheduled at 9:30 a.m., between the Congressional and public tour times. Participants enter at the East Appointment gate. A U.S. Secret Service / Uniformed Division Tour Officer conducts the tour in sign language. Signed tours are available to groups of 8 to 20. Groups are also encouraged to bring their own interpreters.

  Signing interpretation is also available for individual visitors with advance notice. A Congressional office first issues guided tour tickets to a guest who is hearing-impaired and then contacts the Visitors Office at least 2 weeks in advance to request interpreter service.

  The Visitors Office TDD (telephone device for the deaf) is 202-456-2121. Messages may be left outside normal business hours.

  Visually-Impaired

  Tours for visually-impaired groups may be arranged in advance by writing to the Visitors Office, White House, Washington, DC 20502. The tours are usually scheduled at 9:30 a.m., between the Congressional and public tour times. Participants enter at the East Appointment gate. A U.S. Secret Service / Uniformed Division Tour Officer permits visitors to touch specific objects in the House. Touch tours are currently available only to groups of 8 to 20, not to individual visitors. Guide animals are permitted in the White House.

  General Tour Information

  All White House tours are free. Changes in tour schedules are occasionally made because of official events. Notice may not be given until that morning. The Visitors Office 24-hour Information Line recording at 202-456-7041 provides the most up-to-date information. The TDD is 202-456-2121. Visitors should confirm tour schedules by calling the information line the night before and the morning that they plan to visit. It is occasionally necessary to close individual rooms on the tour; however, notice about closed rooms is not possible.

 Prohibited Items

  Prohibited items include, but are not limited to, the following: handbags, book bags, backpacks, purses, food and beverages of any kind, strollers, cameras, video recorders or any type of recording device, tobacco products, personal grooming items (make-up, hair brush or comb, lip or hand lotions, etc.), any pointed objects (pens, knitting needles, etc.), aerosol containers, guns, ammunition, fireworks, electric stun guns, mace, martial arts weapons / devices, or knives of any size. The U.S. Secret Service reserves the right to prohibit any other personal items. Umbrellas, wallets, cell phones and car keys are permitted.

Please note that no storage facilities are available on or around the complex. Individuals who arrive with prohibited items will not be permitted to enter the White House.

  Parking

  The closest Metrorail stations to the White House are Federal Triangle (blue and orange lines), Metro Center (blue, orange, and red lines) and McPherson Square (blue and orange lines). On-street parking is not available near the White House, and use of public transportation is strongly encouraged.

  Restrooms / Public Telephones

  The nearest restrooms and public telephones to the White House are in the Ellipse Visitor Pavilion (the park area south of the White House) and in the White House Visitor Center. Restrooms or public telephones are not available at the White House.

  50. Both Congressional guided and self-guided tours need to be scheduled in advance.

  51. All White House tours are free of charge except on federal holidays.

  52. The White House Visitor Center provides free drinks but not food service.

  53. Wheelchair reservation service is provided by the officer at the Visitors Entrance Building.

  54. Hearing-impaired visitors can request signing interpretation service from the Visitors Office.

  55. Touch tours are currently only offered to visually-impaired groups of 8 to 20.

  56. Sometimes official events make it necessary to close ________________ without notice.

  57. The personal items permitted to be carried into the White House are ________________.

  58. The transportation visitors are encouraged to use is ________________.

  59. Inside the White House, visitors cannot find or use restrooms or ________________.

  Section C (10 marks)

  Directions: In this section, there is one passage followed by 5 questions. Read the passage carefully, then answer the questions in as few words as possible (not more than 10 words). Remember to write the answers on the Answer Sheet.

  Questions 60-64 are based on the following passage.

  If you were on a distant planet, and if you had instruments that could tell you the composition of Earth's atmosphere, how would you know there was life on this planet?

  Water in the atmosphere would suggest there could be water on the surface, and as we all know water is considered crucial to life. But water would only suggest that life is possible. It wouldn't prove it's there.

  Carbon? That basic component of life as we know it? Not necessarily. A diamond is pure carbon, and it may be pretty, but it isn't alive.

  What really sets Earth apart is nitrogen, which makes up 80 percent of the planet's atmosphere. And it's there only because there is abundant life on Earth, say scientists at the University of Southern California.

  The report grew out of a class discussion two years ago in a course taught by Capone and Kenneth Nealson, professor of earth sciences. Students were asked to come up with different ideas about searching for life on other planets. What is a distinct signature,” as Capone puts it, that would show there is life on another planet?

  That's a question that has been kicked around in many quarters in recent decades, especially since all efforts to find some form of life, no matter whether on Mars or in the distant reaches of space, have failed. At least so far.

  The current effort to search for some evidence of life on Mars focuses primarily on the search for water, because it has long been believed that water, or at least some fluid, is necessary for the chemical processes that lead life to take place. But that's probably the wrong approach, the USC group argues.

  “It's hard to imagine life without water, but it's easy to imagine water without life, says Nealson, who was on the Mars team before moving to USC.

  But nitrogen would be a much clearer signature of life. Only about 2 percent to 3 percent of the Martian atmosphere is nitrogen. That's just a trace, and it probably means there is no life on Mars today, and if there was in the past, it probably ended many, many years ago.

  But, the USC team adds quickly, that doesn't mean there's no life anywhere else in the universe. They don't know where, of course, but they may have found a way to narrow down the search. Look first for nitrogen, then look for biological activity that should be there.

  So if life exists elsewhere, and is similar to life as we know it, there should be nitrogen, and that's what we should be looking for first, the researchers say.

  If they don't find nitrogen on Mars, Capone says, that will probably bring us to the conclusion that there likely never was life on Mars.

  But how about elsewhere? Could this technique be used to search for life in other solar systems?

  Maybe. It might be possible to detect a nitrogen-rich atmosphere around a planet orbiting another star, but not yet. Current instruments aren't that sensitive.

  If they ever are, the search for life might be narrowed down to the most promising prospects, chiefly because of the presence of nitrogen. And won't that be fun!

  Questions:

  60. What can suggest life is possible but cannot be proved according to the author?

  61. What is a clear signature of life on another planet according to Capone?

  62. What is considered as a wrong way to search for evidence of life on Mars?

  63. What can probably prove there is no life on Mars today based on the new theory?

  64. Why is it impossible to use the new technique to search for life in other solar systems now?

  Section D (12 marks)

  Directions: In this section, there is one passage followed by a summary. Read the passage carefully and complete the summary below by choosing a maximum of three words from the passage to fill in the spaces 65-70. Remember to write the answers on the Answer Sheet.

  Questions 65-70 are based on the following passage.

  In August 2008, athletes from the United States and around the world will compete in the Beijing Olympics. But did you know that in September of next year, disabled athletes will compete in the Paralympic Games in Beijing?

  The Olympics and the Paralympics are separate movements. But they have always been held in the same year, and since 1988, they have also been held in the same city. The International Olympic Committee and the International Paralympic Committee signed an agreement in 2001 to secure this connection. The next winter games will take place in Vancouver, Canada, in 2010.

  The Paralympic Games grew out of a sports competition held in 1948 in England and a doctor named Ludwig Guttmann organized it for men who suffered spinal cord injuries in World War II. Four years later, it became an international event as competitors from the Netherlands took part. Then, in 1960, the first Paralympics were held in Rome. 400 athletes from 23 countries competed. By 2004, the Paralympic Games in Athens had almost 4000 athletes from 136 countries, who may have physical or mental limitations and may be blind or in wheelchairs. Yet sometimes they perform better than athletes without disabilities.

  In 1968, Eunice Kennedy Shriver, the sister of former President John F. Kennedy, started the Special Olympics, which are just for children and adults with mental limitations and whose programs currently serve more than two million people in 160 countries. In November 2006, in Mumbai, India, teams competed in the First Special Olympics International Cricket Cup. In addition to India, there were men's teams from Afghanistan, Australia, Bangladesh, Nepal, Pakistan, Sri Lanka and the West Indies. There were also women's cricket teams from India and Pakistan.

  There are many organizations in the United States that help people with disabilities play sports. Wheelchair tennis is a popular sport. So is basketball. In fact, there are more than one hundred professional teams playing wheelchair basketball thanks to the special wheelchairs for athletes that are lightweight and designed for quick moves. For people who want to go really fast in their chairs, there is a Power Wheelchair Racing Association.

  In the state of Utah there is a place called the National Ability Center, which teaches all kinds of sports to people with all kinds of physical and mental disabilities and even gives friends and family members a chance to try a sport as if they were disabled.

  A reporter from the Washington Post wanted to know what it would be like for a blind person to use a climbing wall. So, protected by a safety line, the newspaper reporter closed his eyes and started to feel for places to put his hands and feet. Trainers on the ground urged him on: “Take your time. You can do it.” Finally he reached the top.

  At the National Ability Center people can learn to ride horses and mountain bikes. They can try winter mountain sports, and learn scuba diving and other water activities. The center also prepares athletes for the Paralympics.

  These days, the first place many people go when they want to travel is the Internet, where they can get information about hotels, transportation and services like tour companies. The Internet can also help travelers find special services for the disabled. For example, there are groups that help young people with disabilities travel to different countries.

  Susan Sygall, who uses a wheelchair herself, leads an organization called Mobility International USA, and has traveled to more than twenty-five countries to talk about the rights of people with disabilities. She says people with disabilities are all members of a global family and working together across borders is the most powerful way of making changes.

Summary:

The Olympics and the Paralympics are (65) ________ but they have always been held in the same year and also in the same city since 1988 when the International Olympic Committee and the International Paralympic Committee signed an agreement in 2001 to (66) ________. The Paralympic Games grew out of a sports competition organized by a doctor named (67) ________ in 1948 in England for men injured in World War II. In 1952, it became an (68)________ and in 1960, the first Paralympics were held in Rome for people who may have physical or mental limitations or may be blind or in wheelchairs. The (69) ________ was started in 1968 in the United States by Eunice Kennedy Shriver just for children and adults with mental limitations and to help people with disabilities play sports and enjoy other activities, many (70) ________ are founded, such as the Power Wheelchair Racing Association, the National Ability Center and Mobility International USA.

  Part IV Cloze (15 minutes, 15 marks)

  Section A: There are 5 blanks in the passage. Use the word given on the right side to form a word that fits in each blank. Remember to write the answers on the Answer Sheet.

Maria Callas was one of the best-known opera singers in the world, who became famous internationally for her beautiful voice and intense (71)________ during the 1950s, and the recordings of her singing the well-known operas remain very popular today.

Maria Callas was born in New York City in 1923 and her real name was Maria Kalogeropoulous. Her parents were Greek and when she was fourteen, she and her mother returned to Greece, where Maria studied singing at the national conservatory in Athens and the well-known opera (72)________ Elvira de Hidalgo chose Maria as her student.

In 1941, when she was 17, Maria Callas was paid to sing in a major opera for the first time. She sang the (73)________ role in several operas in Athens during the next three years. In 1943, Callas was invited to perform in Italy, which was the real beginning of her profession as an opera singer. She performed major parts in several of the most (74)________ operas. In 1949, she married an Italian (75)________, Giovanni Battista Meneghini, who was twenty years older and became her adviser and manager. person

sing

lead

fame

industry

  Section B: There are 10 blanks in the passage. For each blank, some letters of the word has been given (not exceeding 3 letters). Read the passage below and think of the word which best fits each blank. Use only one word in each blank. Remember to write the answers on the Answer Sheet.

Autism is a general (76)________ for a group of brain disorders that limit the development of social and communication skills, t m

which (77)________ professionals call autism spectrum disorders. me l

Experts say autism is permanent and cannot be cured. But

there are ways to treat it that they say can (78) ________ the re e

severity, and the academy says the earlier treatment begins,

the(79)________ the results. b r

The medical group released two reports Monday with detailed

information to help doctors (80)________autism. Chris Johnson id y

at the University of Texas Health Science Center in San Antonio was one of the authors, who says doctors should look for signs

of autism when they (81)________ babies at eighteen months and ex e

twenty-four months.

Doctors traditionally (82)________ the possibility of co r

autism only if a child shows delayed (83)________ or unusually sp h

repetitive behaviors. These may be clear signs of it, but they usually do not appear until a child is two or three years old.

Parents could answer a list of written questions about

their baby, and then the doctor could (84) ________ tests as pe m

simple as observing the baby's ability to follow a moving object with its eyes. Experts say failing to watch a moving object may be a sign of autism.

Doctors and parents can also look for behaviors that are normal in babies under one year of age. For example, does the

baby appear to (85)________ to a parent's voice? Does the baby re d

make eye contact? Does the baby wave or point at things?

  Part V Translation (15 minutes, 15 marks)

  Section A (8 marks)

  Directions: Translate the underlined sentences of the following passage into Chinese. Remember to write the answers on the Answer Sheet.

  When you talk about China and India's seemingly unstoppable growth, nobody's surprised. After all, the increasing economic strength of the two countries has dominated the news for the past few years. But both China and India face some substantial demographic hurdles to continued expansion. Despite the billion-plus citizens of each country, both may simply lack sufficient qualified workers.

  (86) China is a rapidly aging society whose current challenges of unemployment and overpopulation will, within a decade, shift to different problems: labor shortages and an elderly population with too few children. In fact, China may be the first country to go gray before it reaches developed status. The World Bank estimates that by 2020 the mainland will face a lack of even unskilled labor due to aging.

  China is already facing a shortage of skilled labor. Construction sites lack welders, skilled machine operators, and plumbers. And a recent report said the country is short some 750,000 managers. Despite a 95% literacy rate among all but the oldest citizens, there are not enough well-educated Chinese. (87) This is in part due to a lack of schools that combine basic theory with practical skills and a focus on passing the elite university exams. So despite rising salaries, many of those entering China's workforce cannot learn the skills they need.

  India seems to have an age advantage, with half its population under 25. In the long run, this gives them the upper hand. But at present, India shares China's problem of an insufficiently educated workforce. Citigroup reports that India's talent pool isn't deep enough to meet demand in industries including textiles, aviation, telecom, retail, and engineering.

  By some other measures, both countries look pretty good. China graduates about 1.7 million students from 1,500 colleges and universities annually, of whom 350,000 are engineers. India produces about 3 million college and university graduates, including 440,000 engineers. Those numbers, though, don't tell the whole story.(88) Only 10% to 25% of these graduates are employable by multinationals, and this is only partially because of language challenges, which every human resources executive I've met who works in China or India agrees with.

  The source of the problem is cultural. Young people in these countries are highly motivated to study and learn, but only in theory. There is no tradition of practical application. So engineers don't go into operations, factories, or mines, and don't really know the machines and conditions for which they are designing processes. (89) Chinese and Indian students who have just earned BAs want to pursue MBAs right away, and neither they nor their parents understand the value of practical experience. The degree is seen as a trophy, a traditional ticket to higher status.

  Section B (7 marks)

  Directions: Translate the following sentences into English, using the words given in the brackets. Remember to write your answers on the Answer Sheet.

  90. 如果你告诉他们真相,你就可以获得这次机会而不被他们误解。(without)

  91. 不管多难,我也要及时完成文献翻译。(no matter)

  92. 中国在提高农村居民生活水平方面已经取得了很大成绩,这一事实谁也无法否认。(deny)

  Part VI IQ Test (5 minutes, 5 marks)

  Directions: There are 5 IQ Test questions in this part. Write your answer on the Answer Sheet.

  93. Live is to Evil just as 5423 is to ________.

  94. One half of one quarter of one tenth of 800 is ________.

  95. The number that should come next in the series 64, 16, 4, 1, 1/4 is ________.

  96. Ann is taller than Jill, and Kelly is taller than Ann. Then we can draw the conclusion that Jill is definitely ________ Kelly.

  97. One boy and a girl caught 40 frogs. Jack caught four times as many as Jen did. The number of frogs caught by Jen was ________.

  Part VII Writing (30 minutes, 30 marks)

  Task I (10 marks)

  Directions: You are required to write a Notice in about 100 words to inform the students and English teachers of a lecture on American Liberal Education by Prof. Gilbert in the English Department Hall. Please write it on the Answer Sheet.

  Task II (20 marks)

  Directions: For this part, you are required to write a composition on The Anti-Addiction System. You should write at least 120 words and please write it on the Answer Sheet.

Now in China the anti-addiction system and a “real name checking” system have been adopted in Cyber cafes to prevent players under 18 from becoming addicted to online games. What do students think about this system? What is your opinion about this?

参考答案

Part I Listening Comprehension

Section A 

  1. B 2. A 3. B 4. A 5. C

  Section B

  6. B 7. A 8. C 9. A 10. B 11. C 12. A 13. A 14. B1 5. C

  Section C

  16. B 17. A 18. C 19. C 20. B

  Section D

  21. professor 22. presented 23. brain 24. criminal 25. intelligence tests

  26. removed from 27. water pipes 28. blood test 29. in jail 30. aggression and learning

  Part II Vocabulary and Structure

  31. D 32. A 33. D 34. B 35. A 

  36. D 37. C 38. B 39. C 40. B 

  41. B 42. A4 3. A 44. C 45. D

  Part III Reading Comprehension

  Section A

  46. B47. C48. A49. D

  Section B

  50. Y 51. N 52. NG 53. N 54. Y5 5. Y

  56. individual rooms on the tour

  57. umbrellas, wallets, cell phones and car keys

  58. public transportation / subway

  59. public telephones

  Section C

  60. Water.

  61. Nitrogen. / The presence of nitrogen.

  62. The search for water.

  63. The small amount (2% to 3%) of nitrogen in the Martian atmosphere .

  64. Because current instruments aren't very sensitive.

  Section D

  65. separate movements

  66. secure this connection

  67. Ludwig Guttmann

  68. international event

  69. Special Olympics

  70. organizations

  Part IV Cloze Section A

  71. personality 72. singer 73. leading 74. famous 75. industrialist

  Section B

  76. term 77. medical 78. reduce 79. better 80. identify

  81. examine 82. consider 83. speech 84. perform 85. respond

  Part V Translation

  Section A

  86. 中国正迅速进入老龄化社会,目前存在的失业和人口过剩问题在十年之内将转变成新的问,即劳动力短缺和孩子少老年人多的问题。

  87. 其部分原因是缺少将基础理论和实践技能相结合的学校和过分关注通过高考精英选拔考试。

  88. 只有10%—25%的毕业生有资格被跨国企业雇用,其部分原因就是语言问题;这一点被我所见过的每位在中国或印度工作的人力资源经理都认同。

  89. 中国和印度刚刚取得学士学位的学生就想马上读硕士,而且他们自己以及他们的父母都不理解实践经验的价值。

Section B

  90. If you tell them the truth, you will be able to obtain this opportunity without being misunderstood by them.

  91. I will try to accomplish the translation of the literature in time no matter how hard it is!. 92. Nobody can deny the fact that China has made great achievements in raising living standards of rural residents.

Part VI IQ Test

  93. 3245

  94. 10

  95. 1/16

  96. shorter than

  97. 8

  Part VII Writing

  Task I

  NOTICE

  We are very honored to have Prof. Richard Gilbert from Harvard University to give us a lecture on American Liberal Education. Prof. Gilbert, who graduated from Standford in 1979 and received his Ph.D. in Education from Harvard in 1984, has been conducting research and teaching international students in Harvard for more than 20 years. He has become a leading expert in this field and has published several books and numerous papers on liberal education and education policy. His humorous and thought provoking speech will surely benefit all the audience.

  The lecture will be given in the English Department Conference Hall from 2:30 to 4:30. p.m., Friday afternoon, November 16, 2007. All the teachers and students are welcome.

  English Department

  Task II

  The Anti-Addiction System

  Now in China, the adoption of the anti-addiction system and a real name checking system in netbars has led to a heated debate among students.

  Some say that students' self-discipline is very limited and now at least 14 percent of the urban teenage players are addicted to the Internet and some children even repeatedly disappear from home to spend their nights at local Internet cafes, and play truant from school also. Therefore, they have failed quite a few school exams. In this sense, the new policy can prevent minors developing online addiction.

  Others claim that the anti-addiction system is only a means to an end and cannot remove the root of the online addiction completely and they believe the key is to guide the minors to improve their self-control and balance their work and play.

  In my opinion, the latter point hits home for me because it cuts to the chase. However, education is a long process and takes time, which means before education produces effects on children, some supplementary measures should also be taken to speed up the process and the anti-addiction system is one of them.

2008 National English Contest for College Students

(Level C--- Preliminary)

Part I Listening Comprehension (25 minutes, 30 marks)

Section A (5 marks)

In this section, you will hear five short conversations. At the end of each conversation, a question will be asked about what was said. Both the conversation and the question will be read only once. After each question, there will be a pause. During the pause, read the three choices marked A, B and C, and decide which is the best answer. Then mark the corresponding letter on the Answer Sheet with a single line through the center.

1. A. To buy a newspaper. B. To find a chemist. C. To post a letter.

2. A. Their billing system has been efficient.

B. Their old billing system is outdated.

C. He’s unimpressed with the new billing system.

3. A. She’s been on vacation. B. She’s been at the grocery store.

C. She’s been on a business trip.

4. A. To go to the market. B. To have coffee. C. To have a discussion.

5. A. Changing a flight reservation. B. Canceling a hotel reservation.

C. Reserving an extra hotel room.

Section B (10 marks)

In this section, you will hear two long conversations. Each conversation will be read only once. At the end of each conversation, there will be a one-minute pause. During the pause, read the five questions, each with three choices marked A, B and C, and decide which is the best answer. Then mark the corresponding letter on the Answer Sheet with a single line through the center.

Conversation one

6. What’s Tania’s phone number?

A. 506-6178 B. 501-6678 C. 501-6687

7. Who is going shopping at Harrods?

A. Tania. B. Karl. C. Karl’s mother.

8. How is Tania going to get to Harrods?

A. On foot. B. By taxi. C. By tube.

9. Which street plan shows the way from the station?

A. B. C.

10. What kind of wine does Karl want?

A. Red Merlot. B. White Bordeaux. C. Rose Chianti.

Conversation Two

11. Which drawing shows the position of the handle?

A. B. C.

12. What other product features are included?

A. A range of handle sizes. B. A variety of color combinations.

C. A set of rings for different-sized tins.

13. What’s the purpose of the V-clip?

A. To hold the paint brush so it drips in the can.

B. To prevent paint from getting onto the painted surfaces.

C. To prevent the brush drying out if the painter is interrupted.

14. Who will probably use the can-holder?

A. Artists. B. Decorators. C. Soldiers.

15. How much does the product cost?

A. 4. B. 5. C. $4.

Section C (5 marks)

In this section, you will hear five short news item. Each item will be read only once. After each item, there will be a pause. During the pause, read the question and the three choices marked A, B and C, and decide which is the best answer. Then mark the corresponding letter on the Answer Sheet with a single line through the center.

16. What has prompted the new administration to review the Australia’s citizenship test?

A. It’s out of date. B. Higher immigration rates. C. Many applicants failing it.

17. Which train provides the easiest way to see Mount Fuji?

A. From Tokyo to Yokohama. B. From Osaka to Yokohama.

C. From Tokyo to Osaka.

18. How many U.N. employees were killed in 2007?

A. 17. B. 42. C. 59.

19. What have studies found out about severe pneumonia treatment?

A. No medication works in developing countries.

B. It can be effectively treated at home with oral antibiotics.

C. It can only be treated in the hospital with injectable antibiotics.

20. What was the bar’s business like before the smoking ban?

A. Almost empty every day. B. Usually crowded at midday.

C. Empty on Wednesdays.

Section D (10 marks)

In this section, you will hear a short passage. There are ten missing words or phrases. Fill in the blanks with the exact words you hear on the tape. Remember to write the answers on the Answer Sheet.

Crime isn’t rising because the laws are not strict enough, or because the (21) ________ are not tough enough, or simply because there aren’t enough policemen on the beat. It is rising because the agencies and individuals who can have a (22) _______ effect on crime are not pulling together or in the same (23) ______. The police are one of those agencies, of course. But crime prevention is also a matter for (24) ______ who are content to see mass (25) _______ turn into a way of life, for councils who prefer to shout yah-boo at the police, (26) _____ getting on with making life safer for local people, for architects who design exposed and indefensible buildings, for parents who don’t take enough interest in what their teenage children are getting up to and for (27) ______ who sit inside and moan (28) ______ do anything about the state of the neighborhood or the street. Crime prevention has been neglected for too many by too many. Over the last ten years it has been willfully neglected in defiance of mounting (29) _______ that it is the central hope of (30) ________ action against crime.

Part II Vocabulary and Structures (10 minutes, 15 marks)

There are 15 incomplete sentences in this part. For each blank there are four choices, marked A, B C and D. Choose the one that best completes the sentence, then mark the corresponding letter on the Answer Sheet with a single line through the center.

31. They are looking for _______ man with _______ long dark hair. He is armed and dangerous.

A. a; a B. the; a C. a; / D. /; the

32. It was unfortunate, but she had no _____ but to act as she did.

A. chance. B. opportunity C. option D. solution

33. The _____ driver thinks that accidents only happen to other people.

A. general B. usual C. common D. average

34. How did they manage to steal the Van Gogh? It was right ____ the security guard’s nose.

A. below B. before C. under D. beside

35. The student ______ continuing his studies when he had to return to his home country unexpectedly.

A. is considering B. was considering C. should consider D. has considered

36. My friends and I enjoy doing many of the same things. In that respect, we have a lot _______.

A. in similar B. in particular C. in common D. in accordance

37. Our planned visit to the United States _______ because we were unable to get the visas.

A. fell over B. fell down C. fell through D. fell on

38. Lest anyone _____ it strange, let me assure you that it is quite true.

A. thought B. should think C. had thought D. thinking

39. When Sally ______ the criminal ______ her house, she screamed at the top of her voice because she didn’t want ______.

A. saw; enter; killed B. saw; enter; to be killed

C. sees; entering; be killed D. sees; enter; killing

40. Tina’s children, _____ all live nearby, organized a big party for her eightieth birthday.

A. who B. that C. which D. as

41. Sometimes it is necessary to be careful _______ the right date to sit for a test.

A. when choosing B. when you will choose

C. when you have chosen D. when you chose

42. Einstein’s theory of relativity seemed hard to believe at the time _______.

A. when he first introduced B. that he first introduced it

C. he first introduced D. which he first introduced it

43. ________ really. I like both public transport and driving.

A. On second thoughts B. I could go either way

C. I will never learn to drive D. But then again

44. _____ I’m getting married!

A. Can you keep a secret? B. Well I never!

C. Many happy returns. D. Congratulations!

45. --- So, do you know where you’d like to go?

--- Actually, I’ve heard that it’s very expensive and it’s cold all the time.

A. I beg your pardon? B. Yeah, but it’s not very good.

C. Do you know any good hotels? D. Do you have any suggestions?

Part III Reading Comprehension (20 minutes, 40 marks)

Section A (4 marks)

There is one passage in this section with 4 questions. For each question, there are four choices marked A, B C and D. You should decide on the best choice, then mark the corresponding letter on the Answer Sheet with a single line through the center.

Questions 46-49 are based on the following passage.

Most sore throats are caused by an infection which treatment with antibiotics cannot cure. But with simple remedies the patient normally gets better in 4 or 5 days.

Tonsillitis, however, usually starts with a sore throat which causes pain on swallowing. With children and some adults there may be a fever and the patient is obviously not feeling well. It may be possible to see white spots on the back of the throat. The neck may also swell, both of which are the normal response to infection.

Sometimes a sore throat may occur with the common cold, and with influenza there may be dryness of the throat, pain on coughing and loss of voice.

TREATMENT

Aspirin: To help relieve the pain on swallowing and (if there is one) the fever, use aspirin tablets dissolved in water, so that the patient can gargle before swallowing. Repeat the treatment every 4 hours.

Drink: Encourage the patient to drink plenty of liquids.

Food: Food should not be forced on a patient who does not want to eat.

Steam: If there is pain in the throat on coughing, breathing in steam may help.

CHILDREN

Young children, who may not be able to gargle, should be given aspirin dissolved in water every 4 hours at the right dose for their age.

At one year: a single junior aspirin.

At five years: half an adult aspirin.

At eight years: one whole adult aspirin.

WHEN TO SEE THE DOCTOR

If the sore throat is still getting worse after 2 days.

If the patient complains of earache.

If the patient or parent is very worried.

46. According to the passage, it would appear that most sore throats _________.

A. require an immediate visit to a doctor

B. respond quickly to treatment with an antibiotic

C. rarely turn out to be symptoms of serious illness

D. result in tonsillitis even when treated

47. In order to treat a sore throat one should ________.

A. prevent the patient from eating too much

B. give the patient up to 4 aspirin tablets every day

C. advise the patient to drink large amounts of liquids

D. make the patient gargle with hot liquids

48. You should call the doctor in if _________.

A. swelling occurs in the region of the ears

B. the patient’s condition worsens after 2 days

C. the patient’s throat is still sore after a week

D. the disease spreads to another member of the family

49. What difference is there in the way adults and young children should be treated with aspirin?

A. Young children should not be allowed to gargle with it.

B. Adults should be given tablets to swallow whole.

C. Young children should be given aspirin more often than adults.

D. Adults should be given larger doses of aspirin than children.

Section B (14 marks)

There is one passage in this section with 10 questions. Go over the passage quickly and answer the questions on the Answer Sheet.

For questions 50-55, mark

Y (for YES) if the statement agrees with the information given in the passage;

N (for NO) if the statement contradicts the information given in the passage;

NG (for NOT GIVEN) if the information is not given in the passage.

For questions 56-59, complete the sentences with the information given in the passage.

In 1945 leaders from 51 countries met in San Francisco, California, and organized the United Nations (often called the UN). World War II had just ended. Millions of people had died, and there was destruction everywhere. People hoped they could build a future of world peace through this new organization.

The United Nations has four main goals and purposes:

1. To work together for international peace and to solve international problems;

2. To develop friendly relations among nations;

3. To work together for human rights for everyone of all races, religions, and of both sexes;

4. To build a center where nations can work together for these goals.

Today almost every country in the world is a member of the UN. Each country has signed an agreement that says:

1. All members are equal.

2. All members promise to solve international problems in a peaceful way.

3. No member will use force against another member.

4. All members will help the UN in its actions.

5.The UN will not try to solve problems within countries except to enforce international peace.

Obviously, the United Nations has not been completely successful in its goals. There have been several wars since 1945. However, the organization has helped bring peace to some countries that were at war. It has helped people who left their countries because of wars. It has helped bring independence to colonies.

The main United Nations organization is in New York City, but the UN has a “family” of other related organizations all over the world. These organizations try to provide a better life for everyone. One example is UNICEF, an organization that provides food, medical care, and many other services for poor children wherever they live. Another example is the World Health Organization, which develops medical programs all over the world.

There are thousands of UN workers in developing countries. They work as planners to increase production in farming and industry. They provide medical services, improve education programs, and spread scientific information. They develop programs that provide jobs and better living conditions. They help countries control their population growth.

The United Nations also holds large international conferences, where people meet to discuss important world issues. One conference was about the uses and ownership of oceans. Another was about women. There are also International Years. In these special years, people work together to improve situations and solve problems. One year was the International Year of the Child.

For centuries countries have fought each other, and powerful countries have taken control of weaker ones. It is very difficult to persuade nations to live together in peace. Nevertheless, the United Nations is attempting to do this. It is the only organization that unites the world in the search for peace.

Section C (10 marks)

In this section, there is one passage followed by 5 questions. Read the passage carefully, and then answer the questions in as few words as possible (not more than 10 words). Remember to write the answers on the Answer Sheet.

Questions 60-64 are based on the following passage.

A language is a system of sounds, gestures, or characters used by humans to communicate their ideas and feelings. There are about 4,000 spoken languages in the world. Some are spoken by millions of people. Other languages have only a few speakers.

All languages have rules for forming words and for ordering those words in sentences. In written languages, meaning is expressed through a system of characters and rules for combining those characters. In spoken languages, meaning is expressed through a system of sounds and rules for combining those sounds. Many deaf people use sign languages, in which gestures do the work of the sound system of spoken languages.

Word order is more important in English than it is in some other languages, such as Russian. The sound system is very important in Chinese and in many languages spoken in West Africa.

Languages are always changing, but they change very slowly. People invent new words for their language, borrow words from other languages, and change the meanings of words as needed. For example, the English word byte was invented by computer specialists in 1959. the English word tomato was borrowed from Nahuatl, and American Indian language spoken in Mexico. The English word meat once referred to food in general.

There are several major language families in the world. The languages in each family are related, and scientists think they came from the same parent family. Language families come in different sizes. The Austronesian family contains at least 500 languages, including Pilipino, Malay, and Maori. The Basque language, spoken in northern Spain, is the only member of its language family.

The Indo-European language family contains 55 languages, including English, German, Spanish, Russian, and Hindi.

Another language family is Sino-Tibetan, which includes Chinese, Burmese, and Tibetan. The Afro-Asiatic family includes Arabic, Hebrew, and Amharic. There are about 150 American Indian languages spoken today. These languages have many differences and have been divided into more than 50 language families.

People learn languages by listening, copying what they hear, and using the language. Most children learn their first languages easily, and sometimes other languages as well. Adults often must work harder at learning a second language.

Questions:

Section D (12 marks)

In this section, there is one passage followed by a summary. Read the passage carefully and complete the summary below by choosing a maximum of three words from the passage to fill in each of the spaces, 65-70. Remember to write the answers on the Answer Sheet.

Questions 65-70 are based on the following passage.

Welcome to “Software World” --- bringing you the very latest information on what is currently available on CD-ROM. Are you a director or producer looking for an unusual actor to play a part in a new movie project, or with that special look for a new commercial on TV? OK. The usual procedure would be to contact a theatrical agency, who would try and sell you the idea of using one of the actors listed on their books. Books? Too old-fashioned for you? Then get yourself a copy of this latest electronic database called “The Electronic Curtain”.

The brainchild of casting agent Fred Harkney of the Better Talent Agency, he says he got the idea of an actor’s directory from his son playing computer games. Noting that junior had to type in the details of the characters in his favorite game, he realized he could do the same for the actors he represents. Eventually, he came to include information on nearly three quarters of the approximately 34,000 actors registered and looking for work in Britain. With some theatrical agencies boasting that they represent over 500 performers, the need for this product is not hard to fathom. It can be a nightmare trying to remember just which actor has done what, or just what an actor can do. The database lists details of over 5,500 actors: TV shows they have appeared in, special skills they possess --- everything down to the color of their eyes and other distinguishing physical features. By entering the details of the type of person you are looking for, the database quickly locates only those persons with the particular qualities requested.

One problem though, is that many actors feel it is too impersonal, and they could be missing out on much needed auditions for parts in theatrical shows. On the other hand, it might just get them that elusive job. The days of nervous nail-biting while waiting around to give an audition could well and truly be over. And all because of a small plastic disk. For product details ring this number now: 0171-379-60000.

Summary:

Part IV Cloze (15 minutes, 15 marks)

Section A (5 marks)

There are 5 blanks in the passage. Use the word given on the right-hand side to form a word that fits in each blank. Remember to write the answers on the Answer Sheet.

Section B (10 marks)

There are 10 blanks in the passage. For each blank, one or more letters of the word have been given (not exceeding 3 letters). Read the passage below and decide which word best fits each blank, using only one word in each word in each blank. Remember to write the answers on the Answer Sheet.

True relaxation is most certainly not a matter of flopping down in front of the television with a welcome drink. Nor is it about drifting into an exhausted sleep. Useful though these responses to tension and over-tiredness (76) mi_______ be, we should distinguish between them and conscious relaxation in (77) te_____ of quality and effect. Regardless of the level of tiredness, real relaxation is a state of alertness, yet, at the same (78) t______, passive awareness, in which our bodies are (79) a________ rest while our minds are awake.

Moreover, it is as natural for a healthy person to be relaxed when moving as resting. (80) Be_____ relaxed in action means we bring the appropriate energy to everything we do, (81) W______ needs to be rediscovered is conscious relaxation. With this in (85) m______ we must apply ourselves to understanding stress and the nature of its causes, however deep-seated.

Part V Translation (15 minutes, 15 marks)

Section A (8 marks)

Translate the underlined sentences of the following passage into Chinese. Remember to write the answers on the Answer Sheet.

After my mother passed away, my dad tried even harder to stay healthy and active. Every morning, until the weather turned too cold, he swam in the turquoise pool in the complex where he lived. (86) Each day, no matter how he felt, he swam one more lap than the previous day, just to prove there was always room for improvement. Every few days he reported the new number of laps to me, pride edging his voice.

By his late seventies, in spite of swimming and working six days a week, my dad had noticeably dwindled in strength and energy. By age eighty-one he was in poor health and had to retire. (87) He pretended he didn’t need to lean heavily on me for support as we walked slowly, and I pretended not to notice. His mind was clear, but congestive heart problems and disabling arthritis had worn him down. One day he said, “In case of an emergency, I do not wish to be kept alive by any extraordinary means. I’ve signed an official paper to this effect.” He smiled his wonderful, broad grin and said, “I’ve been blessed to have had your mother as my wife and you as my only child, and I’m ready to go.”

Less than a month later he had a heart attack. In the emergency room, he again reminded his doctor and me of his wishes, but I couldn’t imagine, in spite of this latest crisis, that he wouldn’t always be saying, “Have I told you yet today that I adore you?”

(88) He was miserable in intensive care; tubes seemed to come from every opening. But my dad still had his sense of humor, asking me, “Does this mean we can’t keep our lunch date tomorrow?” His voice faltered.

“I’ll be here to pick you up and we’ll go someplace special,” I answered, a lump in my throat.

(89) Dad refused to look at me for the first time in his life and turned toward the blank green wall next to his hospital bed. There was a painful silence between us. He said, “I don’t want you to remember me like this. Promise me you won’t, darling! And please go now --- I’m so miserable.

Section B (7 marks)

Translate the following sentences into English, using the words given in the brackets. Remember to write the answers on the Answer Sheet.

90. 她在阳光下只晒了一会儿,脸就开始变红了。(exposure)

91. 我情愿你没有对任何人提及此事。(would rather)

92. 采纳新规则的建议是主席提出的。(suggestion, 同位语从句)

Part VI IQ Test (5 minutes, 5 marks)

There are 5 IQ questions in this part. Write the answers on the Answer Sheet.

93. Find the missing number.

7 9 8 10 9 ?

94. Find a nine-letter word by moving from letter to letter in any direction. Each letter must only be used once.

A N I D

K B R U

O C L D

Z M T E

Clue: OK to drink.

95. Complete the sequence.

A C A E A G A K A M A ?

96. How may different arrangements can you make of all the seven letters in the word COMBINE?

97. Change just one letter in each word to find a familiar phrase.

MALE FOOD

Part VII Writing (30 minutes, 30 marks)

Task I (10 marks)

You are planning to take a computer course with a friend who has sent you a letter with a school advertisement. Read the letter and the advertisement, together with the notes. Then write to the school, asking for the information which your friend suggests and adding any relevant questions of your own.

Write a letter of between 80 and 100 words in an appropriate style on the Answer Sheet. Do not write any address.

Task II (20 marks)

Write about the following topic:

Do you agree or disagree? Give reasons for your answer and include any relevant examples from your own knowledge or experience. Write between 100 and 120 words on the Answer Sheet.

参考答案

Part I Listening Comprehension (25 minutes, 30 marks)

Section A (5 marks)

  1. B 2. B 3. A 4. C 5. C

  Section B (10 marks)

  6. B 7. A 8. C 9. C 10. A 11. A 12. C 13. A 14. B 15. A

  Section C (5 marks)

  16. C 17. C 18. B 19. B 20. B

  Section D (10 marks)

  21. sentences 22. preventative 23. direction 24. governments 25. unemployment

  26. instead of 27. householders 28. rather than 29. evidence 30. effective

  Part II Vocabulary and Structures (10 minutes, 15 marks)

  31. C 32. C 33. D 34. C 35. B 36. C 37. C 38. B 39. B 40. A

  41. A 42. B 43. B 44. A 45. D

  Part III Reading Comprehension (20 minutes, 40 marks)

  Section A (4 marks)

  46. C 47. C 48. B 49. D

  Section B (14 marks)

  (注:50-55题,每题1分;56-59题,每题2分。)

  50. Y 51. Y 52. NG 53. N 54. NG 55. N

  56. will use force 57. completely successful 58. large international conferences

  59. attempting to persuade

  Section C (10 marks)

  60. About 4,000. 61. For forming words and for ordering those words in sentences.

  62. By gestures. 63. Word order. 64. The Basque language.

  Section D (12 marks)

  65. three quarters of 66. theatrical agencies 67. TV shows 68. particular qualities

  69. too impersonal 70. product details

Part IV Cloze (15 minutes, 15 marks)

Section A (5 marks)

  71. imperfect 72. selection 73. Similarly 74. confidence 75. honesty

  Section B (10 marks)

  76. might 77. terms 78. time 79. at 80. Being 81. so 82. rather 83. consequence

  84. What 85. mind

  Part V Translation (15 minutes, 15 marks)

  Section A (8 marks)

  86. 不论感觉怎样,他每天都比前一天多游一圈,以此证明每次都有提高的余地。

  87. 一起慢走时父亲装出一副不十分需要我搀扶的样子,而我也佯装不见。

  88. 重病房内他十分痛苦;皮管几乎插遍每个出气的地方。

  89. 父亲今生第一次不愿看我,他把脸转向病床边空荡荡的绿墙。

  Section B (7 marks)

  (注:90-91题,每题2分;92题,3分。)

  90. After only a short exposure to sunlight her face began to turn red.

  91. I would rather you didn't mention this matter to anyone else.

  92. The suggestion that the new rule be adopted came from the chairman.

  Part VI IQ Test (5 minutes, 5 marks)

  93. 11 94. DRINKABLE 95. Q 96. 5040 97. MAKE GOOD

  Part VII Writing (30 minutes, 30 marks)

  Task I (10 marks)

  参考范文略

  Task II (20 marks)

  参考范文略

2009 National English Contest for College Students

(Level C - Preliminary)

参考答案及作文评分标准

Part I Listening Comprehension (25 minutes, 30 marks)

Section A (5 marks)

1. B 2. B 3. C 4. A 5. C

Section B (10 marks)

6. A 7. B 8. C 9. C 10. B 11. A 12. C 13. B 14. A 15. C

Section C (5 marks)

16. C 17. B 18. C 19. B 20. C

Section D (10 marks)

21. losing 22. minimise 23. expenditures 24. vocational 25. available 26. previously 27. complex 28. insurance 29. security and freedom 30. heavily involved

Part II Vocabulary and Structure (10 minutes, 15 marks)

31. A 32. D 33. B 34. C 35. D 36. C 37. B 38. C 39. D 40. C 41. C 42. B 43. D 44. B 45. B

Part III Reading Comprehension (20 minutes, 35 marks)

Section A (5 marks)

46. B 47. A 48. A 49. D 50. D

Section B (10 marks)

51. the family name 52. surprising 53. economic reasons 54. dowries 55. individuals

Section C (10 marks)

56. 2009.

57. It’s a professional body representing managers and personnel staff.

58. Survival.

59. Find new ways to motivate their employees to perform.

60. To help preserve staff loyalty and engagement.

Section D (10 marks)

61. average 62. has been connected / is connected 63. skin 64. nutrients 65. crucial

Part IV Cloze (15 minutes, 15 marks)

66. speaking 67. encouraged 68. cheapest 69. composer 70. between 71. especially

72. like / including 73. hundreds 74. when 75. and 76. belief 77. exploration

78. developing / designing 79. program(me) 80. over

Part V Translation (15 minutes, 20 marks)

Section A (10 marks)

81. 从皇帝身上华丽的龙袍到当今的时装,刺绣给我们的文化和生活增添了许多乐趣。

82. 直到后来,随着国家经济的发展,刺绣才进入平常百姓的生活。

83. 马王堆汉墓出土的文物是这个时期刺绣空前发展的最好见证。

84. 汉语“刺绣”一词中的“绣”,意思是由五种颜色组成的图画或刺绣,它暗示着美丽壮观的事物。

85. 倾注了智慧和热情,刺绣确实是大家闺秀用来打发时间的一种很好的消遣。

Section B (10 marks)

86. She was disabled from birth, but never did she give in to any difficulty.

87. Whenever you say anything, you must at least make yourself understood.

88. It is important for a business to keep pace with changes in the market.

89. If you book your trip through a travel agency, you’re likely to get a big discount.

90. Had we invested in the telecommunications industry, we would be quite rich by now.

Part VI IQ Test (5 minutes, 5 marks)

91. G.

92. Four spades.

93. QZQDS.

94. E resort.

95. ZN.

Part VII Writing (30 minutes, 30 marks)

参考范文略。

Task I 应用文评分标准

一、评分原则:

1. 本题满分为10分。

2. 评分时,先根据文章的内容和语言初步确定其所属档次,然后以该档次的要求来衡量、确定或调整本档次,最后给分。

3. 词数少于80的,从总分中减去2分。

4. 如书写较差,以致影响阅卷,将分数降低一个档次。

二、各档次的给分范围和要求:

第四档:8—10

完全符合写作格式的要求,覆盖所有内容要点,表达思想清楚,文字通顺,连贯性很好,基本上无词汇和语法错误。

第三档:6—7

基本符合写作格式的要求,有个别地方表达思想不够清楚,文字基本通顺、连贯,有少量词汇和语法错误。

第二档:3—5

未恰当完成写作格式的要求,漏掉内容要点,表达思想不清楚,文字多处出现词汇和语法错误,影响了对写作内容的理解。

第一档:1—2

未完成写作格式的要求,明显遗漏主要内容,表达思想紊乱,有较多词汇和语法的重大错误,未能将信息传达给读者。

0

白卷;作文与题目毫不相关;内容太少,无法评判;所写内容无法看清。

Task II 议论文评分标准

一、评分原则:

1. 本题满分为20分。

2. 评分时,先根据文章的内容和语言初步确定其所属档次,然后以该档次的要求来衡量、确定或调整本档次,最后给分。

3. 词数少于120的,从总分中减去2分。

4. 如书写较差,以致影响交流,将分数降低一个档次。

二、各档次的给分范围和要求:

第四档:16—20

完全符合写作格式的要求,覆盖所有内容要点,表达思想清楚,文字通顺,连贯性很好,基本上无词汇和语法错误。

第三档:11—15

基本符合写作格式的要求,有个别地方表达思想不够清楚,文字基本通顺、连贯,有少量词汇和语法错误。

第二档:6—10

未恰当完成写作格式的要求,漏掉内容要点,表达思想不清楚,文字多处出现词汇和语法错误,影响了对写作内容的理解。

第一档:1—5

未完成写作格式的要求,明显遗漏主要内容,表达思想紊乱,有较多词汇和语法的重大错误,未能将信息传达给读者。

0

白卷;作文与题目毫不相关;内容太少,无法评判;所写内容无法看清。

 人类在漫长的岁月里,创造了丰富多彩的音乐文化,从古至今,从东方到西方,中国文化艺术,渊源流长。

  我国最早的歌曲可以追溯到原始社会,例如传说中伏羲时的【网罟之歌】,诗经中的【关关雉鸠】,无论是思想内容,还是艺术形式,都已发展到很高的水平。

  我们华人音乐有着悠久的历史,有着独特的风格,在世界上,希腊的悲剧和喜剧,印度的梵剧和中国的京剧,被称为【世界三大古老戏剧】,而京剧则是国之瑰宝,是我们华人的骄傲,亦是世界上最璀璨的一颗明珠。

  你可知道高山流水遇知音的故事?你可知道诸葛亮身居空城,面对敌兵压境,饮酒抚琴的故事?

  列宁曾经说过:我简直每天都想听奇妙而非凡的音乐,我常常自豪的,也许是幼稚的心情想,人类怎么会创造出这样的奇迹?一个伟大的无产阶级革命家,为什么对音乐如此痴狂?音乐究竟能给我们带来什么?

  泰戈尔说:我举目漫望着各处,尽情的感受美的世界,在我视力所及的地方,充满了弥漫在天地之间的乐曲。

  【二】

  音乐,就是灵魂的漫步,是心事的诉说,是情愫的流淌,是生命在徜徉,它可以让寂寞绽放成一朵花,可以让时光婉约成一首诗,可以让岁月凝聚成一条河,流过山涧,流过小溪,流入你我的麦田……

  我相信所有的人,都曾被一首歌感动过,或为其旋律,或某句歌词,或没有缘由,只是感动,有的时候,我们喜欢一首歌,并不是这首歌有多么好听,歌词写的多么好,而是歌词写的像自己,我们开心的时候听的是音乐,伤心的时候,慢慢懂得了歌词,而真正打动你的不是歌词,而是在你的生命中,关于那首歌的故事……

  或许,在我们每个人的内心深处,都藏着一段如烟的往事,不经阳光,不经雨露,任岁月的青苔覆盖,而突然间,在某个拐角,或者某间咖啡厅,你突然听到了一首歌,或是你熟悉的旋律,刹那间,你泪如雨下,即使你不愿意去回忆,可是瞬间便触碰了你心中最柔软的地方,荡起了心灵最深处的涟漪,这就是音乐的神奇,音乐的魅力!

  【三】

  德国作曲家,维也纳古典音乐代表人贝多芬,49岁时已经完全失聪,然而,他的成名曲【命运交响曲】却是震惊世界,震撼我们的心灵,在他的音乐世界里,你能感受到生命的悲怆,岁月的波澜,和与命运的抗衡,这就是音乐赋予的力量!

  贝多芬说:音乐是比一切智慧、一切哲学更高的启示,谁能渗透我音乐的意义,便能超脱寻常人无以自拔的苦难。

  其实,人生就是一次漫长的旅行,一场艰难的跋涉,无论遇见怎样的风景,繁华过后,终归平淡,无论遇见还是告别,相聚亦是别离,我们都应该怀着感恩的心,善待生命,善待自己……

  每一首歌都是一个故事,每一段音乐都是一段过往,不知哪首歌里写满了你的故事?哪段音乐有你最美的回忆?想念一个人的时候,是否在安静的夜晚?悲伤的时候,是否单曲循环?高兴时分,是否在音乐里手舞足蹈?

  我喜欢音乐,没有任何理由,音乐是我灵魂的伴侣,是我生活的知己,它能懂我的喜,伴我的忧,伴随着淡淡的旋律,它便融入我的生命,浸透我的灵魂。

  我喜欢音乐,音乐不仅仅是一种艺术享受,还能丰富我的生活,给我带来创作灵感,一首歌,或一句歌词,都是我写作的素材,都是我灵感的源泉,它犹如涓涓细流,汩汩流淌,令我思绪翩翩,令我意象浓浓……

  当我忧伤的时候,我喜欢在音乐里漫步,当我快乐的的时候,我喜欢在音乐里起舞,当我迷茫困惑的时候,唯有音乐,才是我最好的陪伴……

  【四】

  红尘喧嚣,世事沧桑,三千烟火,韶光迷离,我们在尘世间行走,凡尘琐事总会困扰于心,我已经习惯了,将浅浅的心事蕴藏在文字里,将淡淡的忧伤释怀在音乐中,委婉的旋律,环绕于耳,凄美的歌词,萦绕于心, 当我累了,倦了,我只想置身于音乐的海洋,忘记凡尘,忘记喧嚣,安静的去听一首歌……



2003-2010年全国大学生英语竞赛C类初赛、决赛真题及答案汇总集(原创)(1)

相关推荐